Patho/Pharm 4 Week 5 Combined

Ace your homework & exams now with Quizwiz!

Which collaborative problem will the nurse include in a care plan for a patient admitted to the hospital with idiopathic aplastic anemia? a. seizures b. infection c. neurogenic shock d.pulmonary edema

ANS: B Because the patient with aplastic anemia has pancytopenia, the patient is at risk for infection and bleeding. There is no increased risk for seizures, neurogenic shock, or pulmonary edema.

A 28-year-old man with von Willebrand disease is admitted to the hospital for minor knee surgery. The nurse will review the coagulation survey to check the a. platelet count. b. bleeding time. c. thrombin time. d. prothrombin time.

ANS: B The bleeding time is affected by von Willebrand disease. Platelet count, prothrombin time, and thrombin time are normal in von Willebrand disease.

The nurse instructs a patient who has osteosarcoma of the tibia about a scheduled above-the-knee amputation. Which patient statement indicates to the nurse the additional teaching is needed? a. "I will need to participate in physical therapy after surgery" b. "I wish I did not need to have chemotherapy after this surgery" c. "I did not have this bone cancer until my leg broke a week ago." d. "I can use the patient-controlled analgesia to manage postoperative pain."

c. "I did not have this bone cancer until my leg broke a week ago."

A patient had an abdominal-perineal resection for colon cancer. Which nursing action is most important to include in the plan of care of the day after surgery? a. teach about a low-residue diet b. monitor output from the stoma c. assess the perineal drainage and incision d. encourage acceptance of the colostomy stoma

c. assess the perineal drainage and incision

d

d

After studying about fungi, which information indicates a correct understanding of fungi? Fungi causing deep or systemic infections: a. are easily treated with penicillin. b. are extremely rare. c. never occur with other infections. d. are commonly opportunistic.

d. are commonly opportunistic.

Which information would indicate more teaching is needed regarding hypersensitivity reactions? Type _______ hypersensitivity reactions involve an antibody response. a. I b. II c. III d. IV

D IV

10. Which information about a patient population would be most useful to help the nurse plan for human immunodeficiency virus (HIV) testing needs? a. Age b. Lifestyle c. Symptoms d. Sexual orientation

a. Age The current Centers for Disease Control and Prevention policy is to offer routine testing for HIV to all individuals age 13 to 64 years. Although lifestyle, symptoms, and sexual orientation may suggest increased risk for HIV infection, the goal is to test all individuals in this age range.

While planning care for a patient experiencing fatigue due to chemotherapy, which of the following is the most appropriate nursing intervention? a. Prioritization and administration of nursing care throughout the day b. Completing all nursing care in the morning so the patient can rest the remainder of the day c. Completing all nursing care in the evening when the patient is more rested d. Limiting visitors, thus promoting the maximal amount of hours for sleep

a. Prioritization and administration of nursing care throughout the day

The following medications are prescribed by the health care provider for a middle-aged patient who uses long-acting morphine (MS Contin) for chronic back pain but still has ongoing pain. Which medication should the nurse question? a. Morphine b. Dexamethasone c. Pentazocine (Talwin) d. Celecoxib (Celebrex)

c. Pentazocine (Talwin) Opioid agonist-antagonists can precipitate withdrawal if used in a patient who is physically dependent on mu agonist drugs such as morphine. The other medications are appropriate for the patient. p. 114

39. The nurse is caring for a patient with left-sided lung cancer. Which finding would be most important for the nurse to report to the health care provider? a. Hematocrit of 32% b. Pain with deep inspiration c. Serum sodium of 126 mEq/L d. Decreased breath sounds on left side

: C The syndrome of inappropriate antidiuretic hormone (and the resulting hyponatremia) is an oncologic metabolic emergency and requires rapid treatment to prevent complications such as seizures and coma. The other findings also require intervention but are common in patients with lung cancer and not immediately life threatening. DIF: Cognitive Level: Analyze (analysis) REF: 263 OBJ: Special Questions: Prioritization

2. The nurse is caring for a patient receiving intravesical bladder chemotherapy. The nurse should monitor for which adverse effect? a. Nausea c. Hematuria b. Alopecia d. Xerostomia

: C The adverse effects of intravesical chemotherapy are confined to the bladder. The other adverse effects are associated with systemic chemotherapy. DIF: Cognitive Level: Apply (application) REF: 252

31. The nurse obtains information about a hospitalized patient who is receiving chemotherapy for colorectal cancer. Which information about the patient alerts the nurse to discuss a possible change in cancer therapy with the health care provider? a. Frequent loose stools b. Nausea and vomiting c. Elevated white blood count (WBC) d. Increased carcinoembryonic antigen (CEA)

: D An increase in CEA indicates that the chemotherapy is not effective for the patient's cancer and may need to be modified. Gastrointestinal adverse effects are common with chemotherapy. The nurse may need to address these, but they would not necessarily indicate a need for a change in therapy. An elevated WBC may indicate infection but does not reflect the effectiveness of the colorectal cancer therapy. DIF: Cognitive Level: Apply (application) REF: 236

32. The nurse reviews the laboratory results of a patient who is receiving chemotherapy. Which laboratory result is most important to report to the health care provider? a. Hematocrit 30% b. Platelets 95,000/μL c. Hemoglobin 10 g/L d. White blood cells (WBC) 2700/μL

: D The low WBC count places the patient at risk for severe infection and is an indication that the chemotherapy dose may need to be lower or that WBC growth factors such as filgrastim (Neupogen) are needed. Although the other laboratory data indicate decreased levels, they do not indicate any immediate life-threatening adverse effects of the chemotherapy. DIF: Cognitive Level: Apply (application) REF: 235 OBJ: Special Questions: Prioritization

: Nursing Process: Diagnosis MSC: NCLEX: Psychosocial Integrity 18. A patient receiving head and neck radiation for larynx cancer has ulcerations over the oral mucosa and tongue and thick, ropey saliva. Which instructions should the nurse give to this patient? a. Remove food debris from the teeth and oral mucosa with a stiff toothbrush. b. Use cotton-tipped applicators dipped in hydrogen peroxide to clean the teeth. c. Gargle and rinse the mouth several times a day with an antiseptic mouthwash. d. Rinse the mouth before and after each meal and at bedtime with a saline solution.

: D The patient should rinse the mouth with a saline solution frequently. A soft toothbrush is used for oral care. Hydrogen peroxide may damage tissues. Antiseptic mouthwashes may irritate the oral mucosa and are not recommended. DIF: Cognitive Level: Apply (application) REF: 251

When a nurse cares for a patient with systemic lupus erythematosus (SLE), the nurse remembers this disease is an example of: a. autoimmunity. b. alloimmunity. c. homoimmunity. d. alleimmunity.

A autoimmunity

When performing discharge teaching for a patient after a vasectomy, the nurse instructs the patient that he a. should continue to use other methods of birth control for 6 weeks. b. should not have sexual intercourse until his 6-week follow-up visit. c. may have temporary erectile dysfunction (ED) because of swelling. d. will notice a decrease in the appearance and volume of his ejaculate.

ANS: A Because it takes about 6 weeks to evacuate sperm that are distal to the vasectomy site, the patient should use contraception for 6 weeks. ED that occurs after vasectomy is psychologic in origin and not related to postoperative swelling. The patient does not need to abstain from intercourse. The appearance and volume of the ejaculate are not changed because sperm are a minor component of the ejaculate.

Which intervention will be included in the nursing care plan for a patient with immune thrombocytopenic purpura (ITP)? a. Assign the patient to a private room. b. Avoid intramuscular (IM) injections. c. Use rinses rather than a soft toothbrush for oral care. d. Restrict activity to passive and active range of motion.

ANS: B IM or subcutaneous injections should be avoided because of the risk for bleeding. A soft toothbrush can be used for oral care. There is no need to restrict activity or place the patient in a private room.

The nurse performing a focused examination to determine possible causes of infertility will assess for a. hydrocele. b. varicocele. c. epididymitis. d. paraphimosis.

ANS: B Persistent varicoceles are commonly associated with infertility. Hydrocele, epididymitis, and paraphimosis are not risk factors for infertility.

30. The nurse assesses a patient who is receiving interleukin-2. Which finding should the nurse report immediately to the health care provider? a. Generalized muscle aches b. Crackles heard at the lung bases c. Complaints of nausea and anorexia d. Oral temperature of 100.6° F (38.1° C)

: B Capillary leak syndrome and acute pulmonary edema are possible toxic effects of interleukin-2. The patient may need oxygen and the nurse should rapidly notify the health care provider. The other findings are common side effects of interleukin-2. DIF: Cognitive Level: Analyze (analysis) REF: 257

4. The nurse should suggest which food choice when providing dietary teaching for a patient scheduled to receive external-beam radiation for abdominal cancer? a. Fruit salad c. Creamed broccoli b. Baked chicken d. Toasted wheat bread

: B Protein is needed for wound healing. To minimize the diarrhea that is commonly associated with bowel radiation, the patient should avoid foods high in roughage, such as fruits and whole grains. Lactose intolerance may develop secondary to radiation, so dairy products should also be avoided. DIF: Cognitive Level: Apply (application) REF: 254

: Nursing Process: Planning MSC: NCLEX: Physiological Integrity 25. The nurse teaches a patient with cancer of the liver about high-protein, high-calorie diet choices. Which snack choice by the patient indicates that the teaching has been effective? a. Lime sherbet c. Fresh strawberries b. Blueberry yogurt d. Cream cheese bagel

: B Yogurt has high biologic value because of the protein and fat content. Fruit salad does not have high amounts of protein or fat. Lime sherbet is lower in fat and protein than yogurt. Cream cheese is low in protein. DIF: Cognitive Level: Apply (application) REF: 261

7. The nurse teaches a patient who is scheduled for a prostate needle biopsy about the procedure. Which statement, if made by the patient, indicates that teaching was effective? a. "The biopsy will remove the cancer in my prostate gland." b. "The biopsy will determine how much longer I have to live." c. "The biopsy will help decide the treatment for my enlarged prostate." d. "The biopsy will indicate whether the cancer has spread to other organs."

: C A biopsy is used to determine whether the prostate enlargement is benign or malignant and determines the type of treatment that will be needed. A biopsy does not give information about metastasis, life expectancy, or the impact of cancer on the patient's life. DIF: Cognitive Level: Apply (application) REF: 238

13. A patient undergoing external radiation has developed a dry desquamation of the skin in the treatment area. The nurse teaches the patient about the management of the skin reaction. Which statement, if made by the patient, indicates the teaching was effective? a. "I can use ice packs to relieve itching." b. "I will scrub the area with warm water." c. "I can buy aloe vera gel to use on my skin." d. "I will expose my skin to a sun lamp each day."

: C Aloe vera gel and cream may be used on the radiated skin area. Ice and sunlamps may injure the skin. Treatment areas should be cleaned gently to avoid further injury. DIF: Cognitive Level: Apply (application) REF: 255

A 30-year-old female complains of fatigue, arthritis, rash, and changes in urine color. Laboratory testing reveals anemia, lymphopenia, and kidney inflammation. Assuming a diagnosis of SLE, which of the following is also likely to be present? a. Anti-LE antibodies b. Autoantibodies c. Antiherpes antibodies d. Anti-CMV antibodies

B Autoantibodies

A 30-year-old male is having difficulty breathing and has been spitting blood. He reports that he began experiencing this reaction after cleaning his pigeons' cages. Testing reveals he is suffering from allergic alveolitis. Which of the following is he experiencing? a. Serum sickness b. Raynaud phenomenon c. Antibody-dependent cytotoxicity d. Arthus reaction

D Arthus reaction

A 5-year-old male presents with low-set ears, a fish-shaped mouth, and involuntary rapid muscular contraction. Laboratory testing reveals decreased calcium levels. Which of the following diagnosis is most likely? a. B-lymphocyte deficiency b. T-lymphocyte deficiency c. Combined immunologic deficiency d. Complement deficiency

b. T-lymphocyte deficiency

The nurse is caring for an unresponsive terminally ill patient who has 20-second periods of apnea followed by periods of deep and rapid breathing. Which action by the nurse would be appropriate? a. Suction the patient's mouth. b. Administer oxygen via face mask. c. Document Cheyne-Stokes respirations. d. Place the patient in high Fowler's position.

c. Document Cheyne-Stokes respirations. Cheyne-Stokes respirations are characterized by periods of apnea alternating with deep and rapid breaths. Cheyne-Stokes respirations are expected in the last days of life and are not position dependent. There is also no need for supplemental oxygen by face mask or suctioning the patient.

When the immunologist says that pathogens possess virulence, what does virulence mean? a. spreads from one individual to others and causes disease b. induces an immune response c. causes disease d. damages tissue

c. causes disease

: Nursing Process: Implementation MSC: NCLEX: Physiological Integrity 16. A chemotherapy drug that causes alopecia is prescribed for a patient. Which action should the nurse take to support the patient's self-esteem? a. Encourage the patient to purchase a wig or hat to wear when hair loss begins. b. Suggest that the patient limit social contacts until regrowth of the hair occurs. c. Teach the patient to wash hair gently with mild shampoo to minimize hair loss. d. Inform the patient that hair usually grows back once chemotherapy is complete.

: A The patient is taught to anticipate hair loss and to be prepared with wigs, scarves, or hats. Limiting social contacts is not appropriate at a time when the patient is likely to need a good social support system. The damage occurs at the hair follicles and will occur regardless of gentle washing or use of a mild shampoo. The information that the hair will grow back is not immediately helpful in maintaining the patient's self-esteem. DIF: Cognitive Level: Apply (application) REF: 256

: Nursing Process: Evaluation MSC: NCLEX: Physiological Integrity 22. Interleukin-2 (IL-2) is used as adjuvant therapy for a patient with metastatic renal cell carcinoma. Which information should the nurse include when explaining the purpose of this therapy to the patient? a. IL-2 enhances the body's immunologic response to tumor cells. b. IL-2 prevents bone marrow depression caused by chemotherapy. c. IL-2 protects normal cells from harmful effects of chemotherapy. d. IL-2 stimulates malignant cells in the resting phase to enter mitosis.

: A IL-2 enhances the ability of the patient's own immune response to suppress tumor cells. IL-2 does not protect normal cells from damage caused by chemotherapy, stimulate malignant cells to enter mitosis, or prevent bone marrow depression. DIF: Cognitive Level: Understand (comprehension) REF: 258

37. The nurse assesses a patient with non-Hodgkin's lymphoma who is receiving an infusion of rituximab (Rituxan). Which assessment finding would require the most rapid action by the nurse? a. Shortness of breath b. Shivering and chills c. Muscle aches and pains d. Temperature of 100.2° F (37.9° C)

: A Rituximab (Rituxan) is a monoclonal antibody. Shortness of breath should be investigated rapidly because anaphylaxis is a possible reaction to monoclonal antibody administration. The nurse will need to rapidly take actions such as stopping the infusion, assessing the patient further, and notifying the health care provider. The other findings will also require action by the nurse, but are not indicative of life-threatening complications. DIF: Cognitive Level: Analyze (analysis) REF: 258 OBJ: Special Questions: Prioritization

5. During a routine health examination, a 40-yr-old patient tells the nurse about a family history of colon cancer. Which action should the nurse take next? a. Obtain more information about the family history. b. Schedule a sigmoidoscopy to provide baseline data. c. Teach the patient about the need for a colonoscopy at age 50. d. Teach the patient how to do home testing for fecal occult blood.

: A The patient may be at increased risk for colon cancer, but the nurse's first action should be further assessment. The other actions may be appropriate, depending on the information that is obtained from the patient with further questioning. DIF: Cognitive Level: Analyze (analysis) REF: 241

33. When caring for a patient who is pancytopenic, which action by unlicensed assistive personnel (UAP) indicates a need for the nurse to intervene? a. The UAP assists the patient to use dental floss after eating. b. The UAP adds baking soda to the patient's saline oral rinses. c. The UAP puts fluoride toothpaste on the patient's toothbrush. d. The UAP has the patient rinse after meals with a saline solution.

: A Use of dental floss is avoided in patients with pancytopenia because of the risk for infection and bleeding. The other actions are appropriate for oral care of a pancytopenic patient. DIF: Cognitive Level: Apply (application) REF: 261 OBJ: Special Questions: Delegation

MULTIPLE RESPONSE 1. The nurse at the clinic is interviewing a 64-yr-old woman who is 5 feet, 3 inches tall and weighs 125 lb (57 kg). The patient has not seen a health care provider for 20 years. She walks 5 miles most days and has a glass of wine two or three times a week. Which topics will the nurse plan to include in patient teaching about cancer screening and decreasing cancer risk (select all that apply)? a. Pap testing b. Tobacco use c. Sunscreen use d. Mammography e. Colorectal screening

: A, C, D, E The patient's age, gender, and history indicate a need for screening and teaching about colorectal cancer, mammography, Pap smears, and sunscreen. The patient does not use tobacco or excessive alcohol, she is physically active, and her body weight is healthy. DIF: Cognitive Level: Analyze (analysis) REF: 235

2. A patient develops neutropenia after receiving chemotherapy. Which information about ways to prevent infection will the nurse include in the teaching plan (select all that apply)? a. Cook food thoroughly before eating. b. Choose low fiber, low residue foods. c. Avoid public transportation such as buses. d. Use rectal suppositories if needed for constipation. e. Talk to the oncologist before having any dental work.

: A, C, E Eating only cooked food and avoiding public transportation will decrease infection risk. A high-fiber diet is recommended for neutropenic patients to decrease constipation. Because bacteria may enter the circulation during dental work or oral surgery, the patient may need to postpone dental work or take antibiotics. DIF: Cognitive Level: Apply (application) REF: 253

34. The nurse supervises the care of a patient with a temporary radioactive cervical implant. Which action by unlicensed assistive personnel (UAP), if observed by the nurse, would require an intervention? a. The UAP flushes the toilet once after emptying the patient's bedpan. b. The UAP stands by the patient's bed for 30 minutes talking with the patient. c. The UAP places the patient's bedding in the laundry container in the hallway. d. The UAP gives the patient an alcohol-containing mouthwash to use for oral care.

: B Because patients with temporary implants emit radioactivity while the implants are in place, exposure to the patient is limited. Laundry and urine and feces do not have any radioactivity and do not require special precautions. Cervical radiation will not affect the oral mucosa, and alcohol-based mouthwash is not contraindicated. DIF: Cognitive Level: Apply (application) REF: 250 OBJ: Special Questions: Delegation

: Nursing Process: Implementation MSC: NCLEX: Physiological Integrity 23. The home health nurse is caring for a patient who has been receiving interferon therapy for treatment of cancer. Which statement by the patient indicates a need for further assessment? a. "I have frequent muscle aches and pains." b. "I rarely have the energy to get out of bed." c. "I experience chills after I inject the interferon." d. "I take acetaminophen (Tylenol) every 4 hours."

: B Fatigue can be a dose-limiting toxicity for use of immunotherapy. Flulike symptoms, such as muscle aches and chills, are common side effects with interferon use. Patients are advised to use acetaminophen every 4 hours. DIF: Cognitive Level: Apply (application) REF: 258

: Nursing Process: Diagnosis MSC: NCLEX: Psychosocial Integrity 28. A hospitalized patient who has received chemotherapy for leukemia develops neutropenia. Which observation by the nurse would indicate a need for further teaching? a. The patient ambulates around the room. b. The patient's visitors bring in fresh peaches. c. The patient cleans with a warm washcloth after having a stool. d. The patient uses soap and shampoo to shower every other day.

: B Fresh, thinned-skin fruits are not permitted in a neutropenic diet because of the risk of bacteria being present. The patient should ambulate in the room rather than the hospital hallway to avoid exposure to other patients or visitors. Because overuse of soap can dry the skin and increase infection risk, showering every other day is acceptable. Careful cleaning after having a bowel movement will help prevent skin breakdown and infection. DIF: Cognitive Level: Apply (application) REF: 253

38. A patient who is being treated for stage IV lung cancer tells the nurse about new-onset back pain. Which action should the nurse take first? a. Give the patient the prescribed PRN opioid. b. Assess for sensation and strength in the legs. c. Notify the health care provider about the symptoms. d. Teach the patient how to use relaxation to reduce pain.

: B Spinal cord compression, an oncologic emergency, can occur with invasion of tumor into the epidural space. The nurse will need to assess the patient further for symptoms such as decreased leg sensation and strength and then notify the health care provider. Administration of opioids or the use of relaxation may be appropriate but only after the nurse has assessed for possible spinal cord compression. DIF: Cognitive Level: Analyze (analysis) REF: 264 OBJ: Special Questions: Prioritization

15. The nurse administers an IV vesicant chemotherapeutic agent to a patient. Which action is most important for the nurse to take? a. Infuse the medication over a short period of time. b. Stop the infusion if swelling is observed at the site. c. Administer the chemotherapy through a small-bore catheter. d. Hold the medication unless a central venous line is available.

: B Swelling at the site may indicate extravasation, and the IV should be stopped immediately. The medication generally should be given slowly to avoid irritation of the vein. The size of the catheter is not as important as administration of vesicants into a running IV line to allow dilution of the chemotherapy drug. These medications can be given through peripheral lines, although central vascular access devices are preferred. DIF: Cognitive Level: Analyze (analysis) REF: 246

12. The nurse is caring for a patient with colon cancer who is scheduled for external radiation therapy to the abdomen. Which information obtained by the nurse would indicate a need for patient teaching? a. The patient has a history of dental caries. b. The patient swims several days each week. c. The patient snacks frequently during the day. d. The patient showers each day with mild soap.

: B The patient is instructed to avoid swimming in salt water or chlorinated pools during the treatment period. The patient does not need to change habits of eating frequently or showering with a mild soap. A history of dental caries will not impact the patient who is scheduled for abdominal radiation. DIF: Cognitive Level: Apply (application) REF: 255

: Nursing Process: Assessment MSC: NCLEX: Physiological Integrity 24. A patient with leukemia is considering whether to have hematopoietic stem cell transplantation (HSCT). The nurse will include which information in the patient's teaching plan? a. Donor bone marrow is transplanted through a sternal or hip incision. b. Hospitalization is required for several weeks after the stem cell transplant. c. The transplant procedure takes place in a sterile operating room to minimize the risk for infection. d. Transplant of the donated cells can be very painful because of the nerves in the tissue lining the bone.

: B The patient requires strict protective isolation to prevent infection for 2 to 4 weeks after HSCT while waiting for the transplanted marrow to start producing cells. The transplanted cells are infused through an IV line so the transplant is not painful, nor is an operating room or incision required. DIF: Cognitive Level: Understand (comprehension) REF: 261

: Nursing Process: Planning MSC: NCLEX: Physiological Integrity 20. A widowed mother of four school-age children is hospitalized with metastatic ovarian cancer. The patient is crying and tells the nurse that she does not know what will happen to her children when she dies. Which response by the nurse is most appropriate? a. "Don't you have any friends that will raise the children for you?" b. "Would you like to talk about options for the care of your children?" c. "For now you need to concentrate on getting well and not worrying about your children." d. "Many patients with cancer live for a long time, so there is time to plan for your children."

: B This response expresses the nurse's willingness to listen and recognizes the patient's concern. The responses beginning "Many patients with cancer live for a long time" and "For now you need to concentrate on getting well" close off discussion of the topic and indicate that the nurse is uncomfortable with the topic. In addition, the patient with metastatic ovarian cancer may not have a long time to plan. Although it is possible that the patient's friends will raise the children, more assessment information is needed before making plans. DIF: Cognitive Level: Apply (application) REF: 265

11. A patient with Hodgkin's lymphoma who is undergoing external radiation therapy tells the nurse, "I am so tired I can hardly get out of bed in the morning." Which intervention should the nurse add to the plan of care? a. Minimize activity until the treatment is completed. b. Establish time to take a short walk almost every day. c. Consult with a psychiatrist for treatment of depression. d. Arrange for delivery of a hospital bed to the patient's home.

: B Walking programs are used to keep the patient active without excessive fatigue. Having a hospital bed does not necessarily address the fatigue. The better option is to stay as active as possible while combating fatigue. Fatigue is expected during treatment and is not an indication of depression. Minimizing activity may lead to weakness and other complications of immobility. DIF: Cognitive Level: Apply (application) REF: 253

9. A patient with a large stomach tumor attached to the liver is scheduled for a debulking procedure. Which information should the nurse teach the patient about the outcome of this procedure? a. Pain will be relieved by cutting sensory nerves in the stomach. b. Relief of pressure in the stomach will promote better nutrition. c. Decreasing the tumor size will improve the effects of other therapy. d. Tumor growth will be controlled by the removal of malignant tissue.

: C A debulking surgery reduces the size of the tumor and makes radiation and chemotherapy more effective. Debulking surgeries do not control tumor growth. The tumor is debulked because it is attached to the liver, a vital organ (not to relieve pressure on the stomach). Debulking does not sever the sensory nerves, although pain may be lessened by the reduction in pressure on the abdominal organs. DIF: Cognitive Level: Understand (comprehension) REF: 245

35. The nurse receives change-of-shift report on the oncology unit. Which patient should the nurse assess first? a. A 35-yr-old patient who has wet desquamation associated with abdominal radiation b. A 42-yr-old patient who is sobbing after receiving a new diagnosis of ovarian cancer c. A 24-yr-old patient who received neck radiation and has blood oozing from the neck d. A 56-yr-old patient who developed a new pericardial friction rub after chest radiation

: C Because neck bleeding may indicate possible carotid artery rupture in a patient who is receiving radiation to the neck, this patient should be seen first. The diagnoses and clinical manifestations for the other patients are not immediately life threatening. DIF: Cognitive Level: Analyze (analysis) REF: 263 OBJ: Special Questions: Multiple Patients

: Nursing Process: Implementation MSC: NCLEX: Psychosocial Integrity 21. A patient who has severe pain associated with terminal pancreatic cancer is being cared for at home by family members. Which finding by the nurse indicates that teaching regarding pain management has been effective? a. The patient uses the ordered opioid pain medication whenever the pain is greater than 5 (0 to 10 scale). b. The patient agrees to take the medications by the IV route in order to improve analgesic effectiveness. c. The patient takes opioids around the clock on a regular schedule and uses additional doses when breakthrough pain occurs. d. The patient states that nonopioid analgesics may be used when the maximal dose of the opioid is reached without adequate pain relief.

: C For chronic cancer pain, analgesics should be taken on a scheduled basis, with additional doses as needed for breakthrough pain. Taking the medications only when pain reaches a certain level does not provide effective pain control. Although nonopioid analgesics may also be used, there is no maximum dose of opioid. Opioids are given until pain control is achieved. The IV route is not more effective than the oral route, and usually the oral route is preferred. DIF: Cognitive Level: Apply (application) REF: 264

: Nursing Process: Evaluation MSC: NCLEX: Physiological Integrity 29. The nurse is caring for a patient diagnosed with stage I colon cancer. When assessing the need for psychologic support, which question by the nurse will provide the most information? a. "How long ago were you diagnosed with this cancer?" b. "Do you have any concerns about body image changes?" c. "Can you tell me what has been helpful to you in the past when coping with stressful events?" d. "Are you familiar with the stages of emotional adjustment to a diagnosis like cancer of the colon?"

: C Information about how the patient has coped with past stressful situations helps the nurse determine usual coping mechanisms and their effectiveness. The length of time since the diagnosis will not provide much information about the patient's need for support. The patient's knowledge of typical stages in adjustment to a critical diagnosis does not provide insight into patient needs for assistance. Because surgical interventions for stage I cancer of the colon may not cause any body image changes, this question is not appropriate at this time. DIF: Cognitive Level: Apply (application) REF: 265

41. After change-of-shift report on the oncology unit, which patient should the nurse assess first? a. Patient who has a platelet count of 82,000/μL after chemotherapy b. Patient who has xerostomia after receiving head and neck radiation c. Patient who is neutropenic and has a temperature of 100.5° F (38.1° C) d. Patient who is worried about getting the prescribed long-acting opioid on time

: C Temperature elevation is an emergency in neutropenic patients because of the risk for rapid progression to severe infections and sepsis. The other patients also require assessments or interventions but do not need to be assessed as urgently. Patients with thrombocytopenia do not have spontaneous bleeding until the platelets are 20,000/μL. Xerostomia does not require immediate intervention. Although breakthrough pain needs to be addressed rapidly, the patient does not appear to have breakthrough pain. DIF: Cognitive Level: Analyze (analysis) REF: 253 OBJ: Special Questions: Prioritization | Special Questions: Multiple Patients

1. A patient who is scheduled for a breast biopsy asks the nurse the difference between a benign tumor and a malignant tumor. Which answer by the nurse is correct? a. "Benign tumors do not cause damage to other tissues." b. "Benign tumors are likely to recur in the same location." c. "Malignant tumors may spread to other tissues or organs." d. "Malignant cells reproduce more rapidly than normal cells."

: C The major difference between benign and malignant tumors is that malignant tumors invade adjacent tissues and spread to distant tissues and benign tumors do not metastasize. The other statements are inaccurate. Both types of tumors may cause damage to adjacent tissues. Malignant cells do not reproduce more rapidly than normal cells. Benign tumors do not usually recur. DIF: Cognitive Level: Understand (comprehension) REF: 240

: Nursing Process: Evaluation MSC: NCLEX: Physiological Integrity 26. A patient with cancer has a nursing diagnosis of imbalanced nutrition: less than body requirements related to altered taste sensation. Which nursing action would address the cause of the patient problem? a. Add protein powder to foods such as casseroles. b. Tell the patient to eat foods that are high in nutrition. c. Avoid giving the patient foods that are strongly disliked. d. Add spices to enhance the flavor of foods that are served.

: C The patient will eat more if disliked foods are avoided and foods that the patient likes are included instead. Additional spice is not usually an effective way to enhance taste. Adding protein powder does not address the issue of taste. The patient's poor intake is not caused by a lack of information about nutrition. DIF: Cognitive Level: Apply (application) REF: 262

8. The nurse teaches a postmenopausal patient with stage III breast cancer about the expected outcomes of cancer treatment. Which patient statement indicates that the teaching has been effective? a. "After cancer has not recurred for 5 years, it is considered cured." b. "The cancer will be cured if the entire tumor is surgically removed." c. "I will need follow-up examinations for many years after treatment before I can be considered cured." d. "Cancer is never cured, but the tumor can be controlled with surgery, chemotherapy, and radiation."

: C The risk of recurrence varies by the type of cancer. Some cancers are considered cured after a shorter time span or after surgery, but stage III breast cancer will require additional therapies and ongoing follow-up. DIF: Cognitive Level: Apply (application) REF: 243

14. A patient with metastatic cancer of the colon experiences severe vomiting after each administration of chemotherapy. Which action, if taken by the nurse, is appropriate? a. Have the patient eat large meals when nausea is not present. b. Offer dry crackers and carbonated fluids during chemotherapy. c. Administer prescribed antiemetics 1 hour before the treatments. d. Give the patient a glass of a citrus fruit beverage during treatments.

: C Treatment with antiemetics before chemotherapy may help prevent nausea. The patient should eat small, frequent meals. Offering food and beverages during chemotherapy is likely to cause nausea. The acidity of citrus fruits may be further irritating to the stomach. DIF: Cognitive Level: Apply (application) REF: 251

: Nursing Process: Implementation MSC: NCLEX: Physiological Integrity 19. A patient has been assigned the nursing diagnosis of imbalanced nutrition: less than body requirements related to painful oral ulcers. Which nursing action will be most effective in improving oral intake? a. Offer the patient frequent small snacks between meals. b. Assist the patient to choose favorite foods from the menu. c. Provide teaching about the importance of nutritional intake. d. Apply prescribed anesthetic gel to oral lesions before meals.

: D Because the etiology of the patient's poor nutrition is the painful oral ulcers, the best intervention is to apply anesthetic gel to the lesions before the patient eats. The other actions might be helpful for other patients with impaired nutrition but would not be as helpful for this patient. DIF: Cognitive Level: Analyze (analysis) REF: 254

3. The nurse is caring for a patient who smokes two packs/day. Which action by the nurse could help reduce the patient's risk of lung cancer? a. Teach the patient about the seven warning signs of cancer. b. Plan to monitor the patient's carcinoembryonic antigen (CEA) level. c. Teach the patient about annual chest x-rays for lung cancer screening. d. Discuss risks associated with cigarette smoking during each patient encounter.

: D Teaching about the risks associated with cigarette smoking is recommended at every patient encounter because cigarette smoking is associated with multiple health problems. The other options may detect lung cancer that is already present but do not reduce the risk. DIF: Cognitive Level: Apply (application) REF: 237

: Nursing Process: Planning MSC: NCLEX: Psychosocial Integrity 17. A patient who has ovarian cancer is crying and tells the nurse, "My husband rarely visits. He just doesn't care." The husband indicates to the nurse that he does not know what to say to his wife. Which nursing diagnosis is appropriate for the nurse to add to the plan of care? a. Compromised family coping related to disruption in lifestyle b. Impaired home maintenance related to perceived role changes c. Risk for caregiver role strain related to burdens of caregiving responsibilities d. Dysfunctional family processes related to effect of illness on family members

: D The data indicate that this diagnosis is most appropriate because poor communication among the family members is affecting family processes. No data suggest a change in lifestyle or its role as an etiology. The data do not support impairment in home maintenance or a burden caused by caregiving responsibilities. DIF: Cognitive Level: Apply (application) REF: 265

: Nursing Process: Implementation MSC: NCLEX: Physiological Integrity 27. During the teaching session for a patient who has a new diagnosis of acute leukemia, the patient is restless and looks away without making eye contact. The patient asks the nurse to repeat the information about the complications associated with chemotherapy. Based on this assessment, which nursing diagnosis is appropriate for the patient? a. Risk for ineffective adherence to treatment related to denial of need for chemotherapy b. Acute confusion related to infiltration of leukemia cells into the central nervous system c. Deficient knowledge: chemotherapy related to a lack of interest in learning about treatment d. Risk for ineffective health maintenance related to possible anxiety about leukemia diagnosis

: D The patient who has a new cancer diagnosis is likely to have high anxiety, which may impact learning and require that the nurse repeat and reinforce information. The patient's history of a recent diagnosis suggests that infiltration of the leukemia is not a likely cause of the confusion. The patient asks for the information to be repeated, indicating that lack of interest in learning and denial are not etiologic factors. DIF: Cognitive Level: Apply (application) REF: 265

36. Which action should the nurse take when caring for a patient who is receiving chemotherapy and complains of problems with concentration? a. Teach the patient to rest the brain by avoiding new activities. b. Teach that "chemo-brain" is a short-term effect of chemotherapy. c. Report patient symptoms immediately to the health care provider. d. Suggest use of a daily planner and encourage adequate rest and sleep.

: D Use of tools to enhance memory and concentration such as a daily planner and adequate rest are helpful for patients who develop "chemo-brain" while receiving chemotherapy. Patients should be encouraged to exercise the brain through new activities. Chemo-brain may be short or long term. There is no urgent need to report common chemotherapy side effects to the provider. DIF: Cognitive Level: Apply (application) REF: 252

During treatment of a patient who has brain cancer, the nurse hears the oncologist mention that the patient has reached the "nadir." The nurse knows that this term means which of these? a. The lowest level of neutrophils reached during therapy. b. The highest level of neutrophils reached during therapy. c. The point at which the adverse effects of chemotherapy will stop. d. The point at which the cytotoxic action against cancer cells is the highest.

A (The lowest neutrophil count reached after a course of chemotherapy is known as the nadir. The other options are incorrect.)

A 21-year-old male athlete admits to using androgenic steroids. The nurse tells him that which of these is a possible adverse effect of these drugs? a. Liver damage b. Renal failure c. Heart failure d. Stevens-Johnson syndrome

A Peliosis of the liver, the formation of blood-filled cavities, is a potential effect of androgenic anabolic steroid therapy and may be life threatening. Other serious hepatic effects are hepatic neoplasms (liver cancer), cholestatic hepatitis, jaundice, and abnormal liver function. The other options are incorrect.

A person is given an attenuated antigen as a vaccine. When the person asks what was given in the vaccine, how should the nurse respond? The antigen is: a. alive, but less infectious. b. mutated, but highly infectious. c. normal, but not infectious. d. inactive, but infectious.

A alive, but less infectious.

The nurse notes in a patient's medication history that the patient is taking the synthetic androgen danazol (Danocrine). Indications for danazol include which conditions? (Select all that apply.) a. Endometriosis b. Decreased sexual libido c. Postpartum breast engorgement d. Fibrocystic breast disease in women e. Hereditary angioedema f. Metastatic breast cancer

A, D, E Danazol is used to treat hereditary angioedema and to treat women who have endometriosis or fibrocystic breast disease. The other options are incorrect.

When giving chemotherapy as cancer treatment, the nurse recognizes that toxicity to rapidly growing normal cells also occurs. Which rapidly growing normal cells are also harmed by chemotherapy? (Select all that apply.) a. Bone marrow cells b. Retinal cells c. Hair follicle cells d. Nerve myelin cells e. Gastrointestinal (GI) mucous membrane cells

A.C.E (Chemotherapy toxicities generally stem from the fact that chemotherapy drugs affect rapidly dividing cells—both harmful cancer cells and healthy, normal cells. Three types of rapidly dividing human cells are the cells of hair follicles, GI tract cells, and bone marrow cells. The other options are incorrect.)

Which information obtained by the nurse assessing a patient admitted with multiple myeloma is most important to report to the health care provider? a. Serum calcium level is 15 mg/dL. b. Patient reports no stool for 5 days. c. Urine sample has Bence-Jones protein. d. Patient is complaining of severe back pain.

ANS: A Hypercalcemia may lead to complications such as dysrhythmias or seizures, and should be addressed quickly. The other patient findings will also be discussed with the health care provider, but are not life threatening.

Which patient statement to the nurse indicates a need for additional instruction about taking oral ferrous sulfate? a. I will call my health care provider if my stools turn black. b. I will take a stool softener if I feel constipated occasionally. c. I should take the iron with orange juice about an hour before eating. d. I should increase my fluid and fiber intake while I am taking iron tablets.

ANS: A It is normal for the stools to appear black when a patient is taking iron, and the patient should not call the doctor about this. The other patient statements are correct.

Which patient should the nurse assign as the roommate for a patient who has aplastic anemia? a. A patient with chronic heart failure b. A patient who has viral pneumonia c. A patient who has right leg cellulitis d. A patient with multiple abdominal drains

ANS: A Patients with aplastic anemia are at risk for infection because of the low white blood cell production associated with this type of anemia, so the nurse should avoid assigning a roommate with any possible infectious process.

A 19-year-old woman with immune thrombocytopenic purpura (ITP) has an order for a platelet transfusion. Which information indicates that the nurse should consult with the health care provider before obtaining and administering platelets? a. The platelet count is 42,000/mL. b. Petechiae are present on the chest. c. Blood pressure (BP) is 94/56 mm Hg. d. Blood is oozing from the venipuncture site.

ANS: A Platelet transfusions are not usually indicated until the platelet count is below 10,000 to 20,000/mL unless the patient is actively bleeding. Therefore the nurse should clarify the order with the health care provider before giving the transfusion. The other data all indicate that bleeding caused by ITP may be occurring and that the platelet transfusion is appropriate.

3. A 51-year-old patient with a small immobile breast lump is considering having a fine-needle aspiration (FNA) biopsy. The nurse explains that an advantage to this procedure is that a. FNA is done in the outpatient clinic and results are available in 1 to 2 days. b. only a small incision is needed, resulting in minimal breast pain and scarring. c. if the biopsy results are negative, no further diagnostic testing will be needed. d. FNA is guided by a mammogram, ensuring that cells are taken from the lesion.

ANS: A FNA is done in outpatient settings and results are available in 24 to 48 hours. No incision is needed. FNA may be guided by ultrasound, but not by mammogram. Because the immobility of the breast lump suggests cancer, further testing will be done if the FNA is negative. DIF: Cognitive Level: Apply (application) TOP: Nursing Process: Implementation MSC: NCLEX: Physiological Integrity

What event occurs in about 70% of the cases of childhood cancers? a. Cured b. Required only chemotherapy c. Participated in clinical trails d. Developed a secondary malignancy

ANS: A More than 70% of children diagnosed with cancer are cured. Some of the factors leading to improved cure rates in pediatric oncology include the use of combination chemotherapy or multimodal treatment for solid childhood tumors and improvements in nursing and supportive care. A partial explanation for the relative lack of progress in curing the adolescent population at the same rate as that realized in the younger pediatric population is the lack of participation in clinical trials. While survivors of childhood cancer are at increased risk of developing a second malignancy later in life, it is not as frequent as 70%. REF: p. 305

A patient with urinary obstruction from benign prostatic hyperplasia (BPH) tells the nurse, "My symptoms are much worse this week." Which response by the nurse is appropriate? a. "Have you taken any over-the-counter (OTC) medications recently?" b. "I will talk to the doctor about a prostate specific antigen (PSA) test." c. "Have you talked to the doctor about surgery such as transurethral resection of the prostate (TURP)?" d. "The prostate gland changes in size from day to day, and this may be making your symptoms worse."

ANS: A Because the patient's increase in symptoms has occurred abruptly, the nurse should ask about OTC medications that might cause contraction of the smooth muscle in the prostate and worsen obstruction. The prostate gland does not vary in size from day to day. A TURP may be needed, but more assessment about possible reasons for the sudden symptom change is a more appropriate first response by the nurse. PSA testing is done to differentiate BPH from prostatic cancer.

The following male patients recently arrived in the emergency department. Which one should the nurse assess first? a. A 19-yr-old patient who is complaining of severe scrotal pain b. A 60-yr-old patient with a nontender ulceration of the glans penis c. A 64-yr-old patient who has dysuria after brachytherapy for prostate cancer d. A 22-yr-old patient who has purulent urethral drainage and severe back pain

ANS: A The patient's age and symptoms suggest possible testicular torsion, which will require rapid treatment to prevent testicular necrosis. The other patients also require assessment by the nurse, but their history and symptoms indicate nonemergent problems (acute prostatitis, cancer of the penis, and radiation-associated urinary tract irritation).

After receiving change-of-shift report for several patients with neutropenia, which patient should the nurse assess first? a. 56-year-old with frequent explosive diarrhea b. 33-year-old with a fever of 100.8 F (38.2 C) c. 66-year-old who has white pharyngeal lesions d. 23-year old who is complaining of severe fatigue

ANS: B Any fever in a neutropenic patient indicates infection and can quickly lead to sepsis and septic shock. Rapid assessment and (if prescribed) initiation of antibiotic therapy within 1 hour are needed. The other patients also need to be assessed but do not exhibit symptoms of potentially life-threatening problems.

A routine complete blood count indicates that an active 80-year-old man may have myelodysplastic syndrome. The nurse will plan to teach the patient about a. blood transfusion b. bone marrow biopsy. c. filgrastim (Neupogen) administration. d. erythropoietin (Epogen) administration.

ANS: B Bone marrow biopsy is needed to make the diagnosis and determine the specific type of myelodysplastic syndrome. The other treatments may be necessary if there is progression of the myelodysplastic syndrome, but the initial action for this asymptomatic patient will be a bone marrow biopsy.

Which information obtained by the nurse caring for a patient with thrombocytopenia should be immediately communicated to the health care provider? a. The platelet count is 52,000/L. b. The patient is difficult to arouse. c. There are purpura on the oral mucosa. d. There are large bruises on the patients back.

ANS: B Difficulty in arousing the patient may indicate a cerebral hemorrhage, which is life threatening and requires immediate action. The other information should be documented and reported but would not be unusual in a patient with thrombocytopenia.

The nurse notes scleral jaundice in a patient being admitted with hemolytic anemia. The nurse will plan to check the laboratory results for the a. Schilling test. b. bilirubin level. c. stool occult blood test. d. gastric analysis testing.

ANS: B Jaundice is caused by the elevation of bilirubin level associated with red blood cell (RBC) hemolysis. The other tests would not be helpful in monitoring or treating a hemolytic anemia.

A patient who is receiving methotrexate for severe rheumatoid arthritis develops a megaloblastic anemia. The nurse will anticipate teaching the patient about increasing oral intake of a. iron b. folic acid. c. cobalamin (vitamin B12). d. ascorbic acid (vitamin C).

ANS: B Methotrexate use can lead to folic acid deficiency. Supplementation with oral folic acid supplements is the usual treatment. The other nutrients would not correct folic acid deficiency, although they would be used to treat other types of anemia.

An appropriate nursing intervention for a hospitalized patient with severe hemolytic anemia is to a. provide a diet high in vitamin K. b. alternate periods of rest and activity. c. teach the patient how to avoid injury. d. place the patient on protective isolation.

ANS: B Nursing care for patients with anemia should alternate periods of rest and activity to encourage activity without causing undue fatigue. There is no indication that the patient has a bleeding disorder, so a diet high in vitamin K or teaching about how to avoid injury is not needed. Protective isolation might be used for a patient with aplastic anemia, but it is not indicated for hemolytic anemia.

It is important for the nurse providing care for a patient with sickle cell crisis to a. limit the patients intake of oral and IV fluids. b. evaluate the effectiveness of opioid analgesics. c. encourage the patient to ambulate as much as tolerated. d. teach the patient about high-protein, high-calorie foods.

ANS: B Pain is the most common clinical manifestation of a crisis and usually requires large doses of continuous opioids for control. Fluid intake should be increased to reduce blood viscosity and improve perfusion. Rest is usually ordered to decrease metabolic requirements. Patients are instructed about the need for dietary folic acid, but high-protein, high-calorie diets are not emphasized.

A patient with a history of a transfusion-related acute lung injury (TRALI) is to receive a transfusion of packed red blood cells (PRBCs). Which action by the nurse will decrease the risk for TRALI for this patient? a. Infuse the PRBCs slowly over 4 hours. b. Transfuse only leukocyte-reduced PRBCs. c. Administer the scheduled diuretic before the transfusion. d. Give the PRN dose of antihistamine before the transfusion.

ANS: B TRALI is caused by a reaction between the donor and the patient leukocytes that causes pulmonary inflammation and capillary leaking. The other actions may help prevent respiratory problems caused by circulatory overload or by allergic reactions, but they will not prevent TRALI.

Which action will the admitting nurse include in the care plan for a 30-year old woman who is neutropenic? a. Avoid any injections. b. Check temperature every 4 hours. c. Omit fruits or vegetables from the diet. d. Place a No Visitors sign on the door.

ANS: B The earliest sign of infection in a neutropenic patient is an elevation in temperature. Although unpeeled fresh fruits and vegetables should be avoided, fruits and vegetables that are peeled or cooked are acceptable. Injections may be required for administration of medications such as filgrastim (Neupogen). The number of visitors may be limited and visitors with communicable diseases should be avoided, but a no visitors policy is not needed.

A 30-year-old man with acute myelogenous leukemia develops an absolute neutrophil count of 850/L while receiving outpatient chemotherapy. Which action by the outpatient clinic nurse is most appropriate? a. Discuss the need for hospital admission to treat the neutropenia. b. Teach the patient to administer filgrastim (Neupogen) injections. c. Plan to discontinue the chemotherapy until the neutropenia resolves. d. Order a high-efficiency particulate air (HEPA) filter for the patients home.

ANS: B The patient may be taught to self-administer filgrastim injections. Although chemotherapy may be stopped with severe neutropenia (neutrophil count less than 500/L), administration of filgrastim usually allows the chemotherapy to continue. Patients with neutropenia are at higher risk for infection when exposed to other patients in the hospital. HEPA filters are expensive and are used in the hospital, where the number of pathogens is much higher than in the patients home environment.

Several patients call the outpatient clinic and ask to make an appointment as soon as possible. Which patient should the nurse schedule to be seen first? a. 44-year-old with sickle cell anemia who says my eyes always look sort of yellow b. 23-year-old with no previous health problems who has a nontender lump in the axilla c. 50-year-old with early-stage chronic lymphocytic leukemia who reports chronic fatigue d. 19-year-old with hemophilia who wants to learn to self-administer factor VII replacement

ANS: B The patients age and presence of a nontender axillary lump suggest possible lymphoma, which needs rapid diagnosis and treatment. The other patients have questions about treatment or symptoms that are consistent with their diagnosis but do not need to be seen urgently.

12. The nurse is providing preoperative teaching about the transverse rectus abdominis musculocutaneous (TRAM) procedure to a patient. Which information will the nurse include? a. Saline-filled implants are placed under the pectoral muscles. b. Recovery from the TRAM surgery takes at least 6 to 8 weeks. c. Muscle tissue removed from the back is used to form a breast. d. TRAM flap procedures may be done in outpatient surgery centers.

ANS: B Patients take at least 6 to 8 weeks to recover from the TRAM surgery. Tissue from the abdomen is used to reconstruct the breast. The TRAM procedure can take up to 8 hours and requires postoperative hospitalization. Saline implants are used in mammoplasty. DIF: Cognitive Level: Apply (application) TOP: Nursing Process: Implementation MSC: NCLEX: Physiological Integrity

Which set of assessment data is consistent for a patient with severe infection that could lead to system failure? a. Blood pressure (BP) 92/52, pulse (P) 56 beats/min, respiratory rate (RR) 10 breaths/min, urine output 1200 mL in past 24 hours b. BP 90/48, P 112 beats/min, RR 26 breaths/min, urine output 240 mL in past 24 hours c. BP 112/64, P 98 beats/min, RR 18 breaths/min, urine output 2400 mL in past 24 hours d. BP 152/90, P 52 beats/min, RR 12 breaths/min, urine output 4800 mL in past 24 hours

ANS: B The patient with severe infection presents with low BP and compensating elevations in pulse to move lower volumes of blood more rapidly and respiration to increase access to oxygen. Urine output decreases to counteract the decreased circulating blood volume and hypotension. These vital signs are all too low: Blood pressure (BP) 92/52, pulse (P) 56 beats/min, respiratory rate (RR) 10 breaths/min, urine output 1200 mL in past 24 hours. The patient with severe infection does have a low BP, but the pulse and respiratory rate increase to compensate. This data is all within normal limits: BP 112/64, P 98 beats/min, RR 18 breaths/min, urine output 2400 mL in past 24 hours. This set of data reflects an elevated BP with a decrease in pulse and respiratory rates along with normal urine output: BP 152/90, P 52 beats/min, RR 12 breaths/min, urine output 4800 mL in past 24 hours. None of these is a typical response to severe infection.

A 27-yr-old patient who has testicular cancer is being admitted for a unilateral orchiectomy. The patient does not talk to his wife and speaks to the nurse only to answer the admission questions. Which action is appropriate for the nurse to take? a. Teach the patient and the wife that impotence is unlikely after unilateral orchiectomy. b. Ask the patient if he has any questions or concerns about the diagnosis and treatment. c. Inform the patient's wife that concerns about sexual function are common with this diagnosis. d. Document the patient's lack of communication on the health record and continue preoperative care.

ANS: B The initial action by the nurse should be assessment for any anxiety or questions about the surgery or postoperative care. The nurse should address the patient, not the spouse, when discussing the diagnosis and any possible concerns. Without further assessment of patient concerns, the nurse should not offer teaching about complications after orchiectomy. Documentation of the patient's lack of interaction is not an adequate nursing action in this situation.

A patient who has benign prostatic hyperplasia (BPH) with urinary retention is admitted to the hospital with elevated blood urea nitrogen (BUN) and creatinine. Which prescribed therapy should the nurse implement first? a. Infuse normal saline at 50 mL/hr. b. Insert a urinary retention catheter. c. Draw blood for a complete blood count. d. Schedule pelvic magnetic resonance imaging

ANS: B The patient data indicate that the patient may have acute kidney injury caused by the BPH. The initial therapy will be to insert a catheter. The other actions are also appropriate, but they can be implemented after the acute urinary retention is resolved.

The nurse will plan to teach the patient scheduled for photovaporization of the prostate (PVP) a. that urine will appear bloody for several days. b. how to care for an indwelling urinary catheter. c. that symptom improvement takes 2 to 3 weeks. d. about complications associated with urethral stenting.

ANS: B The patient will have an indwelling catheter for 24 to 48 hours and will need teaching about catheter care. There is minimal bleeding with this procedure. Symptom improvement is almost immediate after PVP. Stent placement is not included in the procedure.

A patient tells the nurse that he decided to seek treatment for erectile dysfunction (ED) because his wife "is losing patience with the situation." The nurse's follow-up questions should focus on the man's identified concern with a. low self-esteem. b. role performance. c. increased anxiety. d. infrequent intercourse.

ANS: B The patient's statement indicates that the relationship with his wife is his primary concern. Although anxiety, low self-esteem, and ineffective sexuality patterns may also be concerns, the patient information suggests that addressing the role performance problem will lead to the best outcome for this patient.

Which information will the nurse plan to include when teaching a young adult who has a family history of testicular cancer about testicular self-examination? a. Testicular self-examination should be done at least weekly. b. Testicular self-examination should be done in a warm room. c. The only structure normally felt in the scrotal sac is the testis. d. Call the health care provider if one testis is larger than the other.

ANS: B The testes will hang lower in the scrotum when the temperature is warm (e.g., during a shower), and it will be easier to palpate. The epididymis is also normally palpable in the scrotum. One testis is normally larger. Men at high risk should perform testicular self-examination monthly.

When caring for a patient with continuous bladder irrigation after having transurethral resection of the prostate, which action could the nurse delegate to unlicensed assistive personnel (UAP)? a. Teach the patient how to perform Kegel exercises. b. Report any complaints of pain or spasms to the nurse. c. Monitor for increases in bleeding or presence of clots. d. Increase the flow rate of the irrigation if clots are noted.

ANS: B UAP education and role includes reporting patient concerns to supervising nurses. Patient teaching, assessments for complications, and actions such as bladder irrigation require more education and should be done by licensed nursing staff.

A 52-year-old patient has a new diagnosis of pernicious anemia. The nurse determines that the patient understands the teaching about the disorder when the patient states, I a. need to start eating more red meat and liver b. will stop having a glass of wine with dinner. c. could choose nasal spray rather than injections of vitamin B12. d. will need to take a proton pump inhibitor like omeprazole (Prilosec).

ANS: C Because pernicious anemia prevents the absorption of vitamin B12, this patient requires injections or intranasal administration of cobalamin. Alcohol use does not cause cobalamin deficiency. Proton pump inhibitors decrease the absorption of vitamin B12. Eating more foods rich in vitamin B12 is not helpful because the lack of intrinsic factor prevents absorption of the vitamin.

A nurse is discussing preinvasive epithelial tumors of glandular or squamous cell origin. What is the nurse describing? a. Tumor in differentiation b. Dysplastic c. Cancer in situ d. Cancer beyond (meta) situ

ANS: C Early-stage growths that are localized to the epithelium and have not invaded are called cancer in situ. Cancer in situ is early-stage growth and not a tumor in differentiation but is more mature growth. Dysplastic cells do not define cancer in situ.

Which information will the nurse teach a patient who has chronic prostatitis? a. Ibuprofen (Motrin) should provide good pain control. b. Prescribed antibiotics should be taken for 7 to 10 days. c. Intercourse or masturbation will help relieve symptoms. d. Cold packs used every 4 hours will decrease inflammation.

ANS: C Ejaculation helps drain the prostate and relieve pain. Warm baths are recommended to reduce pain. Nonsteroidal antiinflammatory drugs (NSAIDs) are frequently prescribed but usually do not offer adequate pain relief. Antibiotics for chronic prostatitis are taken for 4 to 12 weeks.

The plan of care for a patient immediately after a perineal radical prostatectomy will include decreasing the risk for infection related to a. urinary incontinence. b. prolonged urinary stasis. c. fecal wound contamination. d. suprapubic catheter placement.

ANS: C The perineal approach increases the risk for infection because the incision is located close to the anus, and contamination with feces is possible. Urinary stasis and incontinence do not occur because the patient has a retention catheter in place for 1 to 2 weeks. A urethral catheter is used after the surgery.

A patient who has non-Hodgkins lymphoma is receiving combination treatment with rituximab (Rituxan) and chemotherapy. Which patient assessment finding requires the most rapid action by the nurse? a. Anorexia b. Vomiting c. Oral ulcers d. Lip swelling

ANS: D Lip swelling in angioedema may indicate a hypersensitivity reaction to the rituximab. The nurse should stop the infusion and further assess for anaphylaxis. The other findings may occur with chemotherapy, but are not immediately life threatening.

A critical action by the nurse caring for a patient with an acute exacerbation of polycythemia vera is to a. place the patient on bed rest. b. administer iron supplements. c. avoid use of aspirin products. d. monitor fluid intake and output.

ANS: D Monitoring hydration status is important during an acute exacerbation because the patient is at risk for fluid overload or underhydration. Aspirin therapy is used to decrease risk for thrombosis. The patient should be encouraged to ambulate to prevent deep vein thrombosis (DVT). Iron is contraindicated in patients with polycythemia vera.

Which laboratory result will the nurse expect to show a decreased value if a patient develops heparin-induced thrombocytopenia (HIT)? a. Prothrombin time b. Erythrocyte count c. Fibrinogen degradation products d. Activated partial thromboplastin time

ANS: D Platelet aggregation in HIT causes neutralization of heparin, so that the activated partial thromboplastin time will be shorter and more heparin will be needed to maintain therapeutic levels. The other data will not be affected by HIT.

Following successful treatment of Hodgkins lymphoma for a 55-year-old woman, which topic will the nurse include in patient teaching? a. Potential impact of chemotherapy treatment on fertility b. Application of soothing lotions to treat residual pruritus c. Use of maintenance chemotherapy to maintain remission d. Need for follow-up appointments to screen for malignancy

ANS: D The chemotherapy used in treating Hodgkins lymphoma results in a high incidence of secondary malignancies; follow-up screening is needed. The fertility of a 55-year-old woman will not be impacted by chemotherapy. Maintenance chemotherapy is not used for Hodgkins lymphoma. Pruritus is a clinical manifestation of lymphoma, but should not be a concern after treatment.

24. A patient has had left-sided lumpectomy (breast-conservation surgery) and an axillary lymph node dissection. Which nursing intervention is appropriate to delegate to a licensed practical/vocational nurse (LPN/LVN)? a. Teaching the patient how to avoid injury to the left arm b. Assessing the patients range of motion for the left arm c. Evaluating the patients understanding of instructions about drain care d. Administering an analgesic 30 minutes before scheduled arm exercises

ANS: D LPN/LVN education and scope of practice include administration and evaluation of the effects of analgesics. Assessment, teaching, and evaluation of a patients understanding of instructions are more complex tasks that are more appropriate to RN level education and scope of practice. DIF: Cognitive Level: Apply (application) OBJ: Special Questions: Delegation TOP: Nursing Process: Planning MSC: NCLEX: Safe and Effective Care Environment

A patient who has been diagnosed with stage 2 prostate cancer chooses the option of active surveillance. The nurse will plan to a. vaccinate the patient with sipuleucel-T (Provenge). b. provide the patient with information about cryotherapy. c. teach the patient about placement of intraurethral stents. d. schedule the patient for annual prostate-specific antigen testing.

ANS: D Patients who opt for active surveillance need to have annual digital rectal examinations and prostate-specific antigen testing. Vaccination with sipuleucel-T, cryotherapy, and stent placement are options for patients who choose to have active treatment for prostate cancer.

After a transurethral resection of the prostate (TURP), a 64-yr-old patient with continuous bladder irrigation complains of painful bladder spasms. The nurse observes clots in the urine. Which action should the nurse take first? a. Increase the flow rate of the bladder irrigation. b. Administer the prescribed IV morphine sulfate. c. Give the patient the prescribed belladonna and opium suppository. d. Manually instill and then withdraw 50 mL of saline into the catheter.

ANS: D The assessment suggests that obstruction by a clot is causing the bladder spasms, and the nurse's first action should be to irrigate the catheter manually and to try to remove the clots. IV morphine will not decrease the spasm, although pain may be reduced. Increasing the flow rate of the irrigation will further distend the bladder and may increase spasms. The belladonna and opium suppository will decrease bladder spasms but will not remove the obstructing blood clot.

Which information about continuous bladder irrigation will the nurse teach to a patient who is being admitted for a transurethral resection of the prostate (TURP)? a. Bladder irrigation decreases the risk of postoperative bleeding. b. Hydration and urine output are maintained by bladder irrigation. c. Antibiotics are infused continuously through the bladder irrigation. d. Bladder irrigation prevents obstruction of the catheter after surgery.

ANS: D The purpose of bladder irrigation is to remove clots from the bladder and prevent obstruction of the catheter by clots. The irrigation does not decrease bleeding or improve hydration. Antibiotics are given by the IV route, not through the bladder irrigation.

A patient who has received chemotherapy has a steadily decreasing white blood cell count. The chemotherapy will end on Tuesday afternoon. The oncologist has mentioned that a colony-stimulating factor will be started soon. The nurse knows that the appropriate time to start this medication is when? A. While the patient is still receiving chemotherapy B. Two hours after the chemotherapy ends C. Wednesday afternoon, 24 hours after the chemotherapy ends D. In 2 to 4 days, after the white blood cells have reached their nadir

C

A patient who has been on methotrexate therapy is experiencing mild pain. The patient is asking for aspirin for the pain. The nurse recognizes that which of these is true in this situation? a. The aspirin will aggravate diarrhea. b. The aspirin will masks signs of infection. c. Aspirin can lead to methotrexate toxicity. d. The aspirin will cause no problems for the patient on methotrexate.

C (Methotrexate interacts with weak organic acids, such as aspirin, and can lead to toxicity by displacing the methotrexate from protein-binding sites.)

A 15-year-old male suffers from severe hemorrhage following a motor vehicle accident. He is given a blood transfusion, but shortly afterward the red blood cells are destroyed by agglutination and lysis. Which of the following blood type transfusion type matches would cause this? a. A-A b. B-O c. AB-O d. A-AB

D A-AB

A 40-year-old female is diagnosed with SLE. Which of the following findings would be considered a symptom of this disease? a. Gastrointestinal ulcers b. Decreased glomerular filtration rate c. Rash on trunk and extremities d. Photosensitivity

D Photosensitivity

A 70-year-old retired nurse is interested in nondrug, mind-body therapies, self-management, and alternative strategies to deal with joint discomfort from rheumatoid arthritis. What options should you consider in her plan of care considering her expressed wishes? A. Stationary exercise bicycle, free weights, and spinning class B. Chamomile tea and IcyHot gel C. Acupuncture and attending church services D. Mind-body therapies such as music therapy, distraction techniques, meditation, prayer, hypnosis, guided imagery, relaxation techniques, and pet therapy

D. Mind-body therapies such as music therapy, distraction techniques, meditation, prayer, hypnosis, guided imagery, relaxation techniques, and pet therapy

A patient is not certain whether she and her family should participate in a genetic screening plan. She asks the nurse why the X-linked recessive disorder that has been noted in some of her family members is expressed in males more frequently than in females. What is the nurse's best response? a. "The disease tends to show up in males because they do not have a second X chromosome to balance the expression of the gene." b. "One X chromosome of a pair is always inactive in females. This inactivity effectively negates the effects of the gene." c. "Females are known to have more effective DNA repair mechanisms than males, thus negating the damage caused by the recessive gene." d. "Expression of genes from the male's Y chromosome does not occur in females, so they are essentially immune to the effects of the gene."

a. "The disease tends to show up in males because they do not have a second X chromosome to balance the expression of the gene."

A patient who has just started taking sustained-release morphine sulfate (MS Contin) for chronic arthritic joint pain after a traumatic injury complains of nausea and abdominal fullness. Which action should the nurse take initially? a. Administer the ordered antiemetic medication. b. Order the patient a clear liquid diet until the nausea decreases. c. Tell the patient that the nausea should subside in about a week. d. Consult with the health care provider about using a different opioid.

a. Administer the ordered antiemetic medication. Nausea is frequently experienced with the initiation of opioid therapy, and antiemetics usually are prescribed to treat this expected side effect. The best choice would be to administer the antiemetic medication so the patient can eat. There is no indication that a different opioid is needed, although if the nausea persists, the health care provider may order a change of opioid. Although tolerance develops and the nausea will subside in about a week, it is not appropriate to allow the patient to continue to be nauseated. A clear liquid diet may decrease the nausea but may not provide needed nutrients for injury healing. p. 114

A nurse assesses a postoperative patient 2 days after chest surgery. What findings indicate that the patient requires better pain management (select all that apply)? a. Confusion b. Hypoglycemia c. Poor cough effort d. Shallow breathing e. Elevated temperature

a. Confusion c. Poor cough effort d. Shallow breathing e. Elevated temperature Inadequate pain control can decrease tidal volume and cough effort, leading to complications such as pneumonia with increases in temperature. Poor pain control may lead to confusion through a variety of mechanism, including hypoventilation and poor sleep quality. Stressors such as pain cause increased release of corticosteroids that can result in hyperglycemia. p. 103

A female patient complains of a "scab that just won't heal" under her left breast. During your conversation, she also mentions chronic fatigue, loss of appetite, and slight cough, attributed to allergies. What is the nurse's best action? a. Continue to conduct a symptom analysis to better understand the patient's symptoms and concerns. b. End the appointment and tell the patient to use skin protection during sun exposure. c. Suggest further testing with a cancer specialist and provide the appropriate literature. d. Tell her to put a bandage on the scab and set a follow-up appointment in one week.

a. Continue to conduct a symptom analysis to better understand the patient's symptoms and concerns.

The nurse admits a terminally ill patient to the hospital. What is the first action that the nurse should complete when planning this patient's care? a. Determine the patient's wishes regarding end-of-life care. b. Emphasize the importance of addressing any family issues. c. Discuss the normal grief process with the patient and family. d. Encourage the patient to talk about any fears or unresolved issues.

a. Determine the patient's wishes regarding end-of-life care. The nurse's initial action should be to assess the patient's wishes at this time. The other actions may be implemented if the patient or the family express a desire to discuss fears, understand the grief process, or address family issues, but they should not be implemented until the assessment indicates that they are appropriate.

18. Eight years after seroconversion, a human immunodeficiency virus (HIV)-infected patient has a CD4+ cell count of 800/μL and an undetectable viral load. What should be included in the plan of care at this time? a. Encourage adequate nutrition, exercise, and sleep. b. Teach about the side effects of antiretroviral agents. c. Explain opportunistic infections and antibiotic prophylaxis. d. Monitor symptoms of acquired immunodeficiency syndrome (AIDS).

a. Encourage adequate nutrition, exercise, and sleep. The CD4+ level for this patient is in the normal range, indicating that the patient is the stage of asymptomatic chronic infection when the body is able to produce enough CD4+ cells to maintain a normal CD4+ count. Maintaining healthy lifestyle behaviors is an important goal in this stage. AIDS and increased incidence of opportunistic infections typically develop when the CD4+ count is much lower than normal. Although the initiation of ART is highly individual, it would not be likely that a patient with a normal CD4+ level would receive ART.

A patient with terminal cancer is being admitted to a family-centered inpatient hospice. The patient's spouse visits daily and cheerfully talks with the patient about wedding anniversary plans for the next year. When the nurse asks about any concerns, the spouse says, "I'm busy at work, but otherwise things are fine." Which provisional nursing diagnosis is appropriate for the patient's spouse? a. Ineffective coping related to lack of grieving b. Anxiety related to complicated grieving process c. Hopelessness related to knowledge deficit about cancer d. Caregiver role strain related to spouse's complex care needs

a. Ineffective coping related to lack of grieving The spouse's behavior and statements indicate the absence of anticipatory grieving, which may lead to impaired adjustment as the patient progresses toward death. The spouse does not appear to feel overwhelmed, hopeless, or anxious.

20. An older adult who takes medications for coronary artery disease and hypertension is newly diagnosed with HIV infection and is starting antiretroviral therapy. Which information will the nurse include in patient teaching? a. Many drugs interact with antiretroviral medications. b. HIV infections progress more rapidly in older adults. c. Less frequent CD4+ level monitoring is needed in older adults. d. Hospice care is available for patients with terminal HIV infection.

a. Many drugs interact with antiretroviral medications. The nurse will teach the patient about potential interactions between antiretrovirals and the medications that the patient is using for chronic health problems. Treatment and monitoring of HIV infection is not affected by age. A patient beginning early ART is not a candidate for hospice. Progression of HIV is not affected by age although it may be affected by chronic disease.

The nurse is caring for a terminally ill patient who is experiencing continuous and severe pain. How should the nurse schedule the administration of opioid pain medications? a. Plan around-the-clock routine administration of analgesics. b. Provide PRN doses of medication whenever the patient requests them. c. Suggest small analgesic doses to avoid decreasing the respiratory rate. d. Offer enough pain medication to keep the patient sedated and unaware of stimuli.

a. Plan around-the-clock routine administration of analgesics. The principles of beneficence and nonmaleficence indicate that the goal of pain management in a terminally ill patient is adequate pain relief even if the effect of pain medications could hasten death. Administration of analgesics on a PRN basis will not provide the consistent level of analgesia the patient needs. Patients usually do not require so much pain medication that they are oversedated and unaware of stimuli. Adequate pain relief may require a dosage that will result in a decrease in respiratory rate.

A patient who is using both a fentanyl (Duragesic) patch and immediate-release morphine for chronic cancer pain develops new-onset confusion, dizziness, and a decrease in respiratory rate. Which action should the nurse take first? a. Remove the fentanyl patch. b. Obtain complete vital signs. c. Notify the health care provider. d. Administer prescribed PRN naloxone

a. Remove the fentanyl patch. The assessment data indicate a possible overdose of opioid. The first action should be to remove the patch. Naloxone administration in a patient who has been chronically using opioids can precipitate withdrawal and would not be the first action. Notification of the health care provider and continued monitoring are also needed, but the patient's data indicate that more rapid action is needed. The respiratory rate alone is an indicator for immediate action before obtaining blood pressure, pulse, and temperature. p. 118

A patient who uses a fentanyl (Duragesic) patch for chronic abdominal pain caused by ovarian cancer asks the nurse to administer the prescribed hydrocodone tablets, but the patient is asleep when the nurse returns with the medication. Which action is best for the nurse to take? a. Wake the patient and administer the hydrocodone. b. Wait until the patient wakes up and reassess the pain. c. Suggest the use of nondrug therapies for pain relief instead of additional opioids. d. Consult with the health care provider about changing the fentanyl (Duragesic) dose.

a. Wake the patient and administer the hydrocodone. Because patients with chronic pain frequently use withdrawal and decreased activity as coping mechanisms for pain, sleep is not an indicator that the patient is pain free. The nurse should wake the patient and administer the hydrocodone. p. 107

A patient in hospice is manifesting a decrease in all body system functions except for a heart rate of 124 beats/min and a respiratory rate of 28 breaths/min. Which statement, if made by the nurse to the patient's family member, is most appropriate? a. "These vital signs will continue to increase until death finally occurs." b. "These vital signs are an expected response now but will slow down later." c. "These vital signs may indicate an improvement in the patient's condition." d. "These vital signs are a helpful response to the slowing of other body systems."

b. "These vital signs are an expected response now but will slow down later." An increase in heart and respiratory rate may occur before the slowing of these functions in a dying patient. Heart and respiratory rate typically slow as the patient progresses further toward death. In a dying patient, high respiratory and pulse rates do not indicate improvement or compensation, and it would be inappropriate for the nurse to indicate this to the family.

The son of a dying patient tells the nurse, "Mother doesn't really respond any more when I visit. I don't think she knows that I am here." Which response by the nurse is appropriate? a. "Cut back your visits for now to avoid overtiring your mother." b. "Withdrawal can be a normal response in the process of dying." c. "Most dying patients don't know what is going on around them." d. "It is important to stimulate your mother so she can't retreat from you."

b. "Withdrawal can be a normal response in the process of dying." Withdrawal is a normal psychosocial response to approaching death. Dying patients may maintain the ability to hear while not being able to respond. Stimulation will tire the patient and is not an appropriate response to withdrawal in this circumstance. Visitors are encouraged to be "present" with the patient, talking softly and making physical contact in a way that does not demand a response from the patient.

8. Which patient would benefit from education about HIV preexposure prophylaxis (PrEP)? a. A 23-yr-old woman living with HIV infection. b. A 52-yr-old recently single woman just diagnosed with chlamydia. c. A 33-yr-old hospice worker who received a needle stick injury 3 hours ago. d. A 60-yr-old male in a monogamous relationship with an HIV-uninfected partner.

b. A 52-yr-old recently single woman just diagnosed with chlamydia. Preexposure prophylaxis (PrEP) is used to prevent HIV infection. Persons who would be good candidates for PrEP include individuals with a recent diagnosis of an STI and those with more than one partner. Individuals who are not on PrEP but who have a recent high-risk exposure (such as a needle stick) would be better candidates for postexposure prophylaxis (PEP). A person in a monogamous relationship with an HIV-uninfected partner is considered low-risk for HIV infection.

A patient who has been diagnosed with inoperable lung cancer and has a poor prognosis plans a trip across the country "to settle some issues with family members." The nurse recognizes that the patient is manifesting which psychosocial response to death? a. Protesting the unfairness of death b. Anxiety about unfinished business c. Fear of having lived a meaningless life d. Restlessness about the uncertainty of prognosis

b. Anxiety about unfinished business The patient's statement indicates that there is some unfinished family business that the patient would like to address before dying. There is no indication that the patient is protesting the prognosis, feels uncertain about the prognosis, or fears that life has been meaningless.

Which action is most important for the nurse to take to ensure culturally competent care for an alert, terminally ill Filipino patient? a. Let the family decide how to tell the patient about the terminal diagnosis. b. Ask the patient and family about their preferences for care during this time. c. Obtain information from Filipino staff members about possible cultural needs. d. Remind family members that dying patients prefer to have someone at the bedside.

b. Ask the patient and family about their preferences for care during this time. Because cultural beliefs may vary among people of the same ethnicity, the nurse's best action is to assess the expectations of both the patient and family. The other actions may be appropriate, but the nurse can only plan for individualized culturally competent care after assessment of this patient and family.

A patient with a deep partial thickness burn has been receiving hydromorphone through patient-controlled analgesia (PCA) for 1 week. The nurse caring for the patient during the previous shift reports that the patient wakes up frequently during the night complaining of pain. What action by the nurse is appropriate? a. Administer a dose of morphine every 1 to 2 hours from the PCA machine while the patient is sleeping. b. Consult with the health care provider about using a different treatment protocol to control the patient's pain. c. Request that the health care provider order a bolus dose of morphine to be given when the patient awakens with pain. d. Teach the patient to push the button every 10 minutes for an hour before going to sleep, even if the pain is minimal.

b. Consult with the health care provider about using a different treatment protocol to control the patient's pain. PCAs are best for controlling acute pain. This patient's history indicates a need for a pain management plan that will provide adequate analgesia while the patient is sleeping. Administering a dose of morphine when the patient already has severe pain will not address the problem. Teaching the patient to administer unneeded medication before going to sleep can result in oversedation and respiratory depression. It is illegal for the nurse to administer the morphine for a patient through PCA. p. 107

Which nursing action could the nurse delegate to unlicensed assistive personnel (UAP) when caring for a patient who is using a fentanyl (Duragesic) patch and a heating pad for treatment of chronic back pain? a. Check the skin under the heating pad. b. Count the respiratory rate every 2 hours. c. Ask the patient whether pain control is effective. d. Monitor sedation using the sedation assessment scale.

b. Count the respiratory rate every 2 hours. Obtaining the respiratory rate is included in UAP education and scope of practice. Assessment for sedation, pain control, and skin integrity requires more education and scope of practice. p. 123

A hospice nurse who has become close to a terminally ill patient is present in the home when the patient dies and feels saddened and tearful as the family members begin to cry. Which action should the nurse take at this time? a. Contact a grief counselor as soon as possible. b. Cry along with the patient's family members. c. Leave the home quickly to allow the family to grieve privately. d. Consider leaving hospice work because patient losses are common.

b. Cry along with the patient's family members. It is appropriate for the nurse to cry and express sadness in other ways when a patient dies, and the family is likely to feel that this is supportive. Contacting a grief counselor, leaving the family to grieve privately, and considering whether hospice continues to be a satisfying place to work are all appropriate actions as well, but the nurse's initial action at this time should be to share the grieving process with the family.

The health care provider orders a patient-controlled analgesia (PCA) machine to provide pain relief for a patient with acute surgical pain who has never received opioids before. Which nursing actions regarding opioid administration are appropriate at this time (select all that apply)? a. Assess for signs that the patient is becoming addicted to the opioid. b. Monitor for therapeutic and adverse effects of opioid administration. c. Emphasize that the risk of some opioid side effects increases over time. d. Teach the patient about how analgesics improve postoperative activity levels. e. Provide instructions on decreasing opioid doses by the second postoperative day.

b. Monitor for therapeutic and adverse effects of opioid administration. d. Teach the patient about how analgesics improve postoperative activity levels. Monitoring for pain relief and teaching the patient about how opioid use will improve postoperative outcomes are appropriate actions when administering opioids for acute pain. Although postoperative patients usually need a decreasing amount of opioids by the second postoperative day, each patient's response is individual. Tolerance may occur, but addiction to opioids will not develop in the acute postoperative period. The patient should use the opioids to achieve adequate pain control, so the nurse should not emphasize the adverse effects. p. 115

A young adult patient with metastatic cancer, who is very close to death, appears restless. The patient keeps repeating, "I am not ready to die." Which action is best for the nurse to take? a. Remind the patient that no one feels ready for death. b. Sit at the bedside and ask if there is anything the patient needs. c. Insist that family members remain at the bedside with the patient. d. Tell the patient that everything possible is being done to delay death.

b. Sit at the bedside and ask if there is anything the patient needs. Staying at the bedside and listening allows the patient to discuss any unresolved issues or physical discomforts that should be addressed. Stating that no one feels ready for death fails to address the individual patient's concerns. Telling the patient that everything is being done does not address the patient's fears about dying, especially because the patient is likely to die soon. Family members may not feel comfortable staying at the bedside of a dying patient, and the nurse should not insist that they remain there.

21. The registered nurse (RN) is caring for a patient who is living with HIV and admitted with tuberculosis. Which task can the RN delegate to unlicensed assistive personnel (UAP)? a. Teach the patient how to dispose of tissues with respiratory secretions. b. Stock the patient's room with the necessary personal protective equipment. c. Interview the patient to obtain the names of family members and close contacts. d. Tell the patient's family members the reason for the use of airborne precautions.

b. Stock the patient's room with the necessary personal protective equipment. A patient diagnosed with tuberculosis would be placed on airborne precautions. Because all health care workers are taught about the various types of infection precautions used in the hospital, the UAP can safely stock the room with personal protective equipment. Obtaining contact information and patient teaching are higher-level skills that require RN education and scope of practice.

A 78-year-old patient has been admitted with benign prostatic hyperplasia. What is most appropriate to include in the nursing plan of care? a. limit fluid intake to no more than 1000 mL/day b. leave a light on in the bathroom during the night c. ask the patient to use a urinal so that urine can be measured d. pad the patient's bed to accommodate overflow incontinence

b. leave a light on in the bathroom during the night

A nurse assesses a patient with chronic cancer pain who is receiving imipramine (Tofranil) in addition to long-acting morphine (MS Contin). Which statement, if made by the patient, indicates to the nurse that the patient is receiving adequate pain control? a. "I'm not anxious during the day." b. "Every night I get 8 hours of sleep." c. "I can accomplish activities without much discomfort." d. "I feel less depressed since I've been taking the Tofranil."

c. "I can accomplish activities without much discomfort." Imipramine is being used in this patient to manage chronic pain and improve functional ability. Although the medication is also prescribed for patients with depression, insomnia, and anxiety, the evaluation for this patient is based on improved pain control and activity level. p. 116

The nurse teaches a student nurse about the action of ibuprofen. Which statement, if made by the student, indicates that teaching was effective? a. "The drug decreases pain impulses in the spinal cord." b. "The drug decreases sensitivity of the brain to painful stimuli." c. "The drug decreases production of pain-sensitizing chemicals." d. "The drug decreases the modulating effect of descending nerves."

c. "The drug decreases production of pain-sensitizing chemicals." Nonsteroidal antiinflammatory drugs (NSAIDs) provide analgesic effects by decreasing the production of pain-sensitizing chemicals such as prostaglandins at the site of injury. Transmission of impulses through the spinal cord, brain sensitivity to pain, and the descending nerve pathways are not affected by NSAIDs. p. 104

The nurse reviews the medication orders for an older patient with arthritis in both hips who reports level 3 (0 to 10 scale) hip pain while ambulating. Which medication should the nurse offer as initial therapy? a. Naproxen 200 mg orally b. Oxycodone 5 mg orally c. Acetaminophen 650 mg orally d. Aspirin (acetylsalicylic acid) 650 mg orally

c. Acetaminophen 650 mg orally Acetaminophen is the best first-choice medication. The principle of "start low, go slow" is used to guide therapy when treating older adults because the ability to metabolize medications is decreased and the likelihood of medication interactions is increased. Nonopioid analgesics are used first for mild to moderate pain, although opioids may be used later. Aspirin and nonsteroidal antiinflammatory drugs are associated with a high incidence of gastrointestinal bleeding in older patients. p. 112

A patient with terminal cancer-related pain and a history of opioid abuse complains of breakthrough pain 2 hours before the next dose of sustained-release morphine sulfate (MS Contin) is due. Which action should the nurse take first? a. Use distraction by talking about things the patient enjoys. b. Suggest the use of alternative therapies such as heat or cold. c. Administer the prescribed PRN immediate-acting morphine. d. Consult with the doctor about increasing the MS Contin dose.

c. Administer the prescribed PRN immediate-acting morphine. The patient's pain requires rapid treatment, and the nurse should administer the immediate-acting morphine. Increasing the MS Contin dose and use of alternative therapies and distraction may also be needed, but the initial action should be to use the prescribed analgesic medications. p. 126

The nurse assesses that a patient receiving epidural morphine has not voided for more than 10 hours. What action should the nurse take initially? a. Place an indwelling urinary catheter. b. Monitor for signs of narcotic overdose. c. Ask if the patient feels the need to void. d. Encourage the patient to drink more fluids.

c. Ask if the patient feels the need to void. Urinary retention is a common side effect of epidural opioids. Assess whether the patient feels the need to void. Because urinary retention is a possible side effect, there is no reason for concern of overdose symptoms. Placing an indwelling catheter requires an order from the health care provider. Usually an in-and-out catheter is performed to empty the bladder if the patient is unable to void because of the risk of infection with an indwelling catheter. Encouraging oral fluids may lead to bladder distention if the patient is unable to void, but might be useful if a patient who is able to void has a fluid deficit. p. 114

The nurse is caring for a patient who had abdominal surgery yesterday and is receiving morphine through patient-controlled analgesia (PCA). What action by the nurse is a priority? a. Assessing for nausea b. Auscultating bowel sounds c. Checking the respiratory rate d. Evaluating for sacral redness

c. Checking the respiratory rate The patient's respiratory rate is the highest priority of care while using PCA medication because of the possible respiratory depression. The other areas also require assessment but do not reflect immediately life-threatening complications. p. 115

14. A patient treated for human immunodeficiency virus (HIV) infection for 6 years has developed fat redistribution to the trunk with wasting of the arms, legs, and face. What recommendation will the nurse give to the patient? a. Review foods that are higher in protein. b. Teach about the benefits of daily exercise. c. Discuss a change in antiretroviral therapy. d. Talk about treatment with antifungal agents.

c. Discuss a change in antiretroviral therapy. A frequent first intervention for metabolic disorders is a change in antiretroviral therapy (ART). Treatment with antifungal agents would not be appropriate because there is no indication of fungal infection. Changes in diet or exercise have not proven helpful for this problem.

The nurse is caring for a patient with lung cancer in a home hospice program. Which action by the nurse is appropriate? a. Discuss cancer risk factors and appropriate lifestyle modifications. b. Teach the patient about the purpose of chemotherapy and radiation. c. Encourage the patient to discuss past life events and their meanings. d. Accomplish a thorough head-to-toe assessment several times a week.

c. Encourage the patient to discuss past life events and their meanings. The role of the hospice nurse includes assisting the patient with the important end-of-life task of finding meaning in the patient's life. Frequent head-to-toe assessments are not needed for hospice patients and may tire the patient unnecessarily. Patients admitted to hospice forego curative treatments such as chemotherapy and radiation for lung cancer. Discussion of cancer risk factors and therapies is not appropriate.

19. Which of these patients who have arrived at the human immunodeficiency virus (HIV) clinic should the nurse assess first? a. Patient whose rapid HIV-antibody test is positive b. Patient whose latest CD4+ count has dropped to 250/μL c. Patient who has had 10 liquid stools in the last 24 hours d. Patient who has nausea from prescribed antiretroviral drugs

c. Patient who has had 10 liquid stools in the last 24 hours The nurse should assess the patient for dehydration and hypovolemia. The other patients also will require assessment and possible interventions, but do not require immediate action to prevent complications such as hypovolemia and shock.

The nurse on a surgical inpatient unit is caring for several patients. Which patient should the nurse assess first? a. Patient with postoperative pain who received morphine sulfate IV 15 minutes ago b. Patient who received hydromorphone (Dilaudid) 1 hour ago and is currently asleep c. Patient who was treated for pain just prior to return from the postanesthesia care unit d. Patient with neuropathic pain who is scheduled to receive a dose of hydrocodone (Lortab) now

c. Patient who was treated for pain just prior to return from the postanesthesia care unit The risk for oversedation is greatest in the first 4 hours after transfer from the postanesthesia care unit. Patients should be reassessed 30 minutes after receiving IV opioids for pain. A scheduled oral medication does not need to be administered exactly at the scheduled time. A patient who falls asleep after pain medication can be allowed to rest. p. 115

22. The nurse designs a program to decrease the incidence of human immunodeficiency virus (HIV) infection in the adolescent and young adult populations. Which information should the nurse assign as the highest priority? a. Methods to prevent perinatal HIV transmission b. Ways to sterilize needles used by injectable drug users c. Prevention of HIV transmission between sexual partners d. Means to prevent transmission through blood transfusions

c. Prevention of HIV transmission between sexual partners Sexual transmission is the most common way that HIV is transmitted. The nurse should also provide teaching about perinatal transmission, needle sterilization, and blood transfusion, but the rate of HIV infection associated with these situations is lower.

3. A patient informed of a positive rapid antibody test result for human immunodeficiency virus (HIV) is anxious and does not appear to hear what the nurse is saying. What action by the nurse is most important at this time? a. Teach the patient how to reduce risky behaviors. b. Inform the patient about the available treatments. c. Remind the patient about the need to return for retesting to verify the results. d. Ask the patient to identify individuals who had intimate contact with the patient.

c. Remind the patient about the need to return for retesting to verify the results. After an initial positive antibody test result, the next step is retesting to confirm the results. A patient who is anxious is not likely to be able to take in new information or be willing to disclose information about the HIV status of other individuals.

After change of shift report, which patient should the nurse assess first? a. a 40 year old with a pleural effusion who reports severe stabbing chest pain b. a 72 year old with cor pulmonale who has 4+ bilateral edema in his legs and feet c. a 64 year old with lung cancer and tracheal deviation after subclavian catheter insertion d. a 28 year old with a history of a lung transplant 1 month ago and a fever of 101F

c. a 64 year old with lung cancer and tracheal deviation after subclavian catheter insertion

Which should the nurse ask when assessing a 60-year-old patient who has a history of benign prostatic hyperplasia? a. "Have you noticed any unusual discharge from your penis?" b. "has there been any change in your sex life in the past year?" c. has there been a decrease in the force of your urinary system?" d. "have you been experiencing difficulty in achieving an erection?"

c. has there been a decrease in the force of your urinary system?"

After studying about viruses, which information indicates the student has a good understanding of viruses? Viruses: a. contain no DNA or RNA. b. are capable of independent reproduction. c. replicate their genetic material inside host cells. d. are easily killed by antimicrobials.

c. replicate their genetic material inside host cells.

The nurse is administering an interferon and will implement which intervention? A. Giving the medication with meals B. Monitoring daily weights C. Limiting fluids while the patient is taking this medication D. Rotating injection sites

d

A 30-year-old male was diagnosed with HIV. Which of the following treatments would be most effective? a.Reverse transcriptase inhibitors b. Protease inhibitors c. Entrance inhibitors d. Antiretroviral therapy (ART)

d. Antiretroviral therapy (ART)

As the nurse admits a patient in end-stage renal disease to the hospital, the patient tells the nurse, "If my heart or breathing stop, I do not want to be resuscitated." Which action should the nurse take first? a. Place a "Do Not Resuscitate" (DNR) notation in the patient's care plan. b. Invite the patient to add a notarized advance directive in the health record. c. Advise the patient to designate a person to make future health care decisions. d. Ask if the decision has been discussed with the patient's health care provider.

d. Ask if the decision has been discussed with the patient's health care provider. A health care provider's order should be written describing the actions that the nurses should take if the patient requires CPR, but the primary right to decide belongs to the patient or family. The nurse should document the patient's request but does not have the authority to place the DNR order in the care plan. A notarized advance directive is not needed to establish the patient's wishes. The patient may need a durable power of attorney for health care (or the equivalent), but this does not address the patient's current concern with possible resuscitation.

A patient who has had good control for chronic pain using a fentanyl (Duragesic) patch reports rapid onset pain at a level 9 (0 to 10 scale) and requests "something for pain that will work quickly." How will the nurse document the type of pain reported by this patient? a. Somatic pain b. Referred pain c. Neuropathic pain d. Breakthrough pain

d. Breakthrough pain Pain that occurs beyond the chronic pain already being treated by appropriate analgesics is termed breakthrough pain. Neuropathic pain is caused by damage to peripheral nerves or the central nervous system. Somatic pain is localized and arises from bone, joint, muscle, skin, or connective tissue. Referred pain is pain that is localized in uninjured tissue. p. 108

In caring for a patient following lobectomy for lung cancer, which of the following should the nurse include in the plan of care? a. Position the patient on the operative side only. b. Avoid administering narcotic pain medications. c. Keep the patient on strict bed rest. d. Instruct the patient to cough and deep breathe.

d. Instruct the patient to cough and deep breathe.

The nurse is completing the medication reconciliation form for a patient admitted with chronic cancer pain. Which medication is of most concern to the nurse? a. Amitriptyline 50 mg at bedtime b. Ibuprofen 800 mg 3 times daily c. Oxycodone (OxyContin) 80 mg twice daily d. Meperidine (Demerol) 25 mg every 4 hours

d. Meperidine (Demerol) 25 mg every 4 hours Meperidine is contraindicated for chronic pain because it forms a metabolite that is neurotoxic and can cause seizures when used for prolonged periods. The ibuprofen, amitriptyline, and oxycodone are appropriate medications for long-term pain management. p. 114

The nurse is caring for a patient who received a bone marrow transplant 10 days ago. The nurse would monitor for which of the following clinical manifestations that could indicate a potentially life-threatening situation? a. Mucositis b. Confusion c. Depression d. Mild temperature elevation

d. Mild temperature elevation

A patient who is receiving sustained-release morphine sulfate (MS Contin) every 12 hours for chronic pain experiences level 9 (0 to 10 scale) breakthrough pain and anxiety. Which action by the nurse is appropriate for treating this change in assessment? a. Suggest amitriptyline 10 mg orally. b. Administer lorazepam (Ativan) 1 mg orally. c. Give ibuprofen (Motrin) 400 to 800 mg orally. d. Offer immediate-release morphine 30 mg orally.

d. Offer immediate-release morphine 30 mg orally. The severe breakthrough pain indicates that the initial therapy should be a rapidly acting opioid, such as the immediate-release morphine. Lorazepam and amitriptyline may be appropriate to use as adjuvant therapy, but they are not likely to block severe breakthrough pain. Use of antianxiety agents for pain control is inappropriate because this patient's anxiety is caused by the pain. p. 108

16. The nurse prepares to administer the following medications to a hospitalized patient with human immunodeficiency (HIV). Which medication is most important to administer at the scheduled time? a. Nystatin tablet b. Oral acyclovir (Zovirax) c. Aerosolized pentamidine (NebuPent) d. Oral tenofovir AF/emtricitabine/bictegravir (Biktarvy)

d. Oral tenofovir AF/emtricitabine/bictegravir (Biktarvy) It is important that antiretrovirals be taken at the prescribed time every day to avoid developing drug-resistant HIV. The other medications should also be given as close as possible to the correct time, but they are not as essential to receive at the same time every day.

A patient who is human immunodeficiency virus (HIV)-infected has a CD4+ cell count of 400/μL. Which factor is most important for the nurse to determine before the initiation of antiretroviral therapy (ART) for this patient? a. CD4+ cell count b. How the patient obtained HIV c. Patient's tolerance for potential medication side effects d. Patient's ability to follow a complex medication regimen

d. Patient's ability to follow a complex medication regimen Drug resistance develops quickly unless the patient takes ART medications on a strict, regular schedule. In addition, drug resistance endangers both the patient and community. The other information is also important to consider, but patients who are unable to manage and follow a complex drug treatment regimen should not be considered for ART.

17. The nurse is caring for a patient who is human immunodeficiency virus (HIV) positive and taking antiretroviral therapy (ART). Which information is most important for the nurse to address when planning care? a. The patient reports feeling "constantly tired." b. The patient reports having no side effects from the medications. c. The patient is unable to explain the effects of atorvastatin (Lipitor). d. The patient reports missing doses of tenofovir AF/emtricitabine (Descovy).

d. The patient reports missing doses of tenofovir AF/emtricitabine (Descovy). Because missing doses of ART can lead to drug resistance, this patient statement indicates the need for interventions such as teaching or changes in the drug scheduling. Fatigue is a common side effect of ART. The nurse should discuss medication actions and side effects with the patient, but this is not as important as addressing the skipped doses of Descovy.

13. A patient with human immunodeficiency virus (HIV) infection has developed Cryptosporidium parvum infection. Which outcome would be appropriate for the nurse to include in the plan of care? a. The patient will be free from injury. b. The patient will receive immunizations. c. The patient will have adequate oxygenation. d. The patient will maintain intact perineal skin.

d. The patient will maintain intact perineal skin. The major manifestation of C. pravum infection is loose, watery stools, which would increase the risk for perineal skin breakdown. The other outcomes would be appropriate for other complications (e.g., pneumonia, dementia, influenza) associated with HIV infection.

Which finding by the nurse will be most helpful in determining whether a 67-year-old patient with benign prostatic hyperplasia has an upper urinary tract infection? a. bladder distention b. foul-smelling urine c. suprapubic discomfort d. costovertebral tenderness

d. costovertebral tenderness

The nurse would correctly respond that the etiology of a congenital immune deficiency is due to a(n): a. negative response to an immunization. b. adverse response to a medication. c. renal failure. d. genetic defect.

d. genetic defect.

A nurse recalls bacteria become resistant to antimicrobials by: a. proliferation. b. attenuation. c. specialization. d. mutation.

d. mutation.

A 63-year-old male patient is scheduled for a physical examination, and he tells the nurse that he wants to start taking a vitamin formula that includes saw palmetto for prostate health. Which is the nurse's best response? a. "I've heard many good things about saw palmetto." b. "It's not a good idea to start herbal therapy at your age." c. "There are very few adverse effects with saw palmetto therapy." d. "The doctor will need to draw some blood and do a digital rectal exam first."

D A prostatic-specific antigen test and digital rectal examination needs to be performed before initiation of treatment with saw palmetto for benign prostatic hyperplasia. Adverse effects may include gastrointestinal upset, headache, back pain, and dysuria.

The nurse notes in a female patient's history that she has an order for the androgen methyltestosterone (Android). Based on this finding, the nurse interprets that the patient has which disorder? a. Fibrocystic breast disease b. Hereditary angioedema c. Hypertension d. Inoperable breast cancer

D Methyltestosterone can be used in cases of inoperable breast cancer in women. The other options are incorrect.

What is the chance that two siblings share both HLA haplotypes, making them a good match for an organ transplant from one to the other? a. 100% b. 75% c. 50% d. 25%

D 25%

A client is experiencing severe pain in the left lower quadrant of the abdomen that is rated as a 10 on a pain scale of 0-10. The client is also experiencing nausea, vomiting, and restlessness. Based on this data, the nurse concludes that the client is experiencing which phenomenon? A. Chronic pain B. Fibromyalgia pain C. End-of-life pain D. Acute pain

D. Acute pain

A patient will be receiving mitoxantrone (Novantrone), 12 mg/m2 every 3 weeks, as part of treatment for prostate cancer. Each dose is mixed into a 50-mL bag of D5W and needs to infuse over 15 minutes. The infusion pump delivers the dose at milliliters per hour. Identify the nurse will set the pump to infuse at what rate. _______

200 mL/hr

A patient is to receive an infusion of 250 mL of platelets over 2 hours through tubing that is labeled 1 mL equals 10 drops. How many drops per minute will the nurse infuse?

21 drops/min

A patient is to receive a daily dose of fludarabine (Fludara), 25 mg/m2 /day for 5 consecutive days. Each dose is diluted in a 125-mL bag of normal saline and is to infuse over 30 minutes. The nurse will set the infusion pump to what rate in milliliters per hour? ______

250 mL/hr

A patient is to receive filgrastim (Neupogen) 5 mcg/kg/day. The patient weighs 198 pounds. Identify how many micrograms of medication this patient will receive each day.

450 mcg

A patient will be receiving aldesleukin [IL-2] (Proleukin), 600,000 IU/kg every 8 hours for 14 doses. The patient weighs 220 pounds. Identify how many IU of medication this patient will receive per dose.

60 million

6. A patient who is diagnosed with cervical cancer classified as Tis, N0, M0 asks the nurse what the letters and numbers mean. Which response by the nurse is accurate? a. "The cancer involves only the cervix." b. "The cancer cells look like normal cells." c. "Further testing is needed to determine the spread of the cancer." d. "It is difficult to determine the original site of the cervical cancer."

: A Cancer in situ indicates that the cancer is localized to the cervix and is not invasive at this time. Cell differentiation is not indicated by clinical staging. Because the cancer is in situ, the origin is the cervix. Further testing is not indicated given that the cancer has not spread. DIF: Cognitive Level: Apply (application) REF: 241

10. External-beam radiation is planned for a patient with cervical cancer. What instructions should the nurse give to the patient to prevent complications from the effects of the radiation? a. Test all stools for the presence of blood. b. Maintain a high-residue, high-fiber diet. c. Clean the perianal area carefully after every bowel movement. d. Inspect the mouth and throat daily for the appearance of thrush.

: C Radiation to the abdomen will affect organs in the radiation path, such as the bowel, and cause frequent diarrhea. Careful cleaning of this area will help decrease the risk for skin breakdown and infection. Stools are likely to have occult blood from the inflammation associated with radiation, so routine testing of stools for blood is not indicated. Radiation to the abdomen will not cause stomatitis. A low-residue diet is recommended to avoid irritation of the bowel when patients receive abdominal radiation. DIF: Cognitive Level: Apply (application) REF: 251

40. An older adult patient who has colorectal cancer is receiving IV fluids at 175 mL/hr in conjunction with the prescribed chemotherapy. Which finding by the nurse is most important to report to the health care provider? a. Patient complains of severe fatigue. b. Patient voids every hour during the day. c. Patient takes only 50% of meals and refuses snacks. d. Patient has crackles up to the midline posterior chest.

: D Rapid fluid infusions may cause heart failure, especially in older patients. The other findings are common in patients who have cancer or are receiving chemotherapy. DIF: Cognitive Level: Analyze (analysis) REF: 266 OBJ: Special Questions: Prioritization

During interleukin drug therapy, a patient is showing signs of severe fluid retention, with increasing dyspnea and severe peripheral edema. The next dose of the interleukin is due now. Which action will the nurse take next? A. Hold the drug, and notify the prescriber. B. Give the drug, and notify the prescriber. C. Give the drug along with acetaminophen and diphenhydramine (Benadryl). D. Monitor the patient for 2 hours, and then give the drug if the patient's condition improves.

A

One patient has cancer of the bone; another has cancer in the connective tissues of the thigh muscles; a third patient has cancer in the vascular tissues. Which of these is the correct term for these tumors? a. Sarcoma b. Leukemia c. Carcinoma d. Lymphoma

A (Sarcomas are malignant tumors that arise from connective tissues. These tissues can be found in bone, cartilage, muscle, blood, lymphatic, and vascular tissues. The other options are incorrect.)

The nurse is monitoring a patient who has severe bone marrow suppression following antineoplastic drug therapy. Which is considered a principal early sign of infection? a. Fever b. Diaphoresis c. Tachycardia d. Elevated white blood cell count

A (Fever and/or chills may be the first sign of an oncoming infection. Elevated white blood cell count will not occur because of the bone marrow suppression. The other options are incorrect.)

The nurse is reviewing the medication list of a patient who will be starting androgen therapy. Which drug classes, if taken with androgens, may have an interaction with them? a. Oral anticoagulants b. Nitrates c. Beta blockers d. Proton pump inhibitors

A Androgens, when used with oral anticoagulants, can significantly increase or decrease anticoagulant activity. The other options are incorrect.

When a male patient is receiving androgen therapy, the nurse will monitor for signs of excessive androgens such as: a. fluid retention. b. dehydration. c. restlessness. d. visual changes.

A Fluid retention is an undesirable effect of androgens. The other options are incorrect.

An immunologist is discussing endotoxin production. Which information should the immunologist include? Endotoxins are produced by: a. gram-negative bacteria. b. gram-positive bacteria. c. gram-negative fungi. d. gram-positive fungi.

A gram-negative bacteria.

Which nursing actions for the care of a dying patient can the nurse delegate to a licensed practical/vocational nurse (LPN/LVN) (select all that apply)? a. Provide postmortem care to the patient. b. Encourage the family members to talk with and reassure the patient. c. Determine how frequently physical assessments are needed for the patient. d. Teach family members about commonly occurring signs of approaching death. e. Administer the prescribed morphine sulfate sublingual as necessary for pain control.

A, B, E a. Provide postmortem care to the patient. b. Encourage the family members to talk with and reassure the patient. e. Administer the prescribed morphine sulfate sublingual as necessary for pain control. Medication administration, psychosocial care, and postmortem care are included in LPN/LVN education and scope of practice. Patient and family teaching and assessment and planning of frequency for assessments are skills that require registered nurse level education and scope of practice.

A 30-year-old female is diagnosed with systemic lupus erythematosus (SLE). Which symptoms are a result of a type II hypersensitivity? a. Anemia b. Seizures c. Lymphopenia d. Facial rash e. Photosensitivity

A, C

The nurse is instructing a male patient about application of transdermal testosterone gel (AndroGel). Which body location is preferred for this medication? (Select all that apply.) a. Back b. Chest c. Thigh d. Scrotum e. Abdomen f. Upper arms

A, C, E, F AndroGel is applied to the skin of the back, abdomen, upper arms, or thighs. Testoderm patches are applied to the scrotal skin.

An 80-year-old male patient is in the intensive care unit has suffered a fractured femur. You are making rounds and notice he is somnolent, with no response to verbal or physical stimulation. He has been on round the clock opioid doses q 4 hours. What is the nurse's first action? A. Call the rapid response team to care for the patient immediately. B. Start a second intravenous line with a large bore catheter. C. Discontinue the opioids on the medication administration record. D. Assess the patient's blood pressure and pain level.

A. Call the rapid response team to care for the patient immediately.

A 62-year-old male has fallen while trimming tree branches sustaining tissue injury. He describes his condition as an aching, throbbing back. This is characteristic of what type of pain? A. Nociceptive pain B. Chronic pain C. Mixed pain syndrome D. Neuropathic pain

A. Nociceptive pain

Methotrexate is ordered for a patient with a malignant tumor, and the nurse is providing education about self-care after the chemotherapy is given. Which statements by the nurse are appropriate for the patient receiving methotrexate? (Select all that apply.) a. Report unusual bleeding or bruising. b. Hair loss is not expected with this drug. c. Prepare for hair loss. d. Avoid areas with large crowds or gatherings. e. Avoid foods that are too hot or too cold or rough in texture. f. Restrict fluid intake to reduce nausea and vomiting.

A.C.D.E (Counsel patients who are taking methotrexate to expect hair loss and to report any unusual bleeding or bruising. Because of the possibility of infection, avoid areas with large crowds or gatherings. Foods that are too hot or too cold or rough in texture may be irritating to the oral mucosa. Fluid intake is to be encouraged to prevent dehydration.)

A patient will be receiving testosterone cypionate (Depo-Testosterone) 400 mg intramuscularly every 4 weeks. The medication is available in a 200-mg/mL strength. Identify how many milliliters will the nurse draw up for each injection. ______

ANS: 2mL

Which action will the nurse include in the plan of care for a 72-year-old woman admitted with multiple myeloma? a. Monitor fluid intake and output. b. Administer calcium supplements. c. Assess lymph nodes for enlargement. d. Limit weight bearing and ambulation.

ANS: A A high fluid intake and urine output helps prevent the complications of kidney stones caused by hypercalcemia and renal failure caused by deposition of Bence-Jones protein in the renal tubules. Weight bearing and ambulation are encouraged to help bone retain calcium. Lymph nodes are not enlarged with multiple myeloma. Calcium supplements will further increase the patients calcium level and are not used.

An appropriate nursing intervention for a patient with non-Hodgkins lymphoma whose platelet count drops to 18,000/L during chemotherapy is to a. check all stools for occult blood. b. encourage fluids to 3000 mL/day. c. provide oral hygiene every 2 hours. d. check the temperature every 4 hours.

ANS: A Because the patient is at risk for spontaneous bleeding, the nurse should check stools for occult blood. A low platelet count does not require an increased fluid intake. Oral hygiene is important, but it is not necessary to provide oral care every 2 hours. The low platelet count does not increase risk for infection, so frequent temperature monitoring is not indicated.

Which menu choice indicates that the patient understands the nurses teaching about best dietary choices for iron-deficiency anemia? a. Omelet and whole wheat toast b. Cantaloupe and cottage cheese c. Strawberry and banana fruit plate d. Cornmeal muffin and orange juice

ANS: A Eggs and whole grain breads are high in iron. The other choices are appropriate for other nutritional deficiencies but are not the best choice for a patient with iron-deficiency anemia.

The nurse caring for a patient with type A hemophilia being admitted to the hospital with severe pain and swelling in the right knee will a. immobilize the joint. b. apply heat to the knee. c. assist the patient with light weight bearing. d. perform passive range of motion to the knee.

ANS: A The initial action should be total rest of the knee to minimize bleeding. Ice packs are used to decrease bleeding. Range of motion (ROM) and weight-bearing exercise are contraindicated initially, but after the bleeding stops, ROM and physical therapy are started.

The nurse has obtained the health history, physical assessment data, and laboratory results shown in the accompanying figure for a patient admitted with aplastic anemia. Which information is most important to communicate to the health care provider? a. Neutropenia b. Increasing fatigue c. Thrombocytopenia d. Frequent constipation

ANS: A The low white blood cell count indicates that the patient is at high risk for infection and needs immediate actions to diagnose and treat the cause of the leucopenia. The other information may require further assessment or treatment, but does not place the patient at immediate risk for complications.

14. Which information will the nurse include in patient teaching for a 36-year-old patient who is scheduled for stereotactic core biopsy of the breast? a. A local anesthetic will be given before the biopsy specimen is obtained. b. You will need to lie flat on your back and lie very still during the biopsy. c. A thin needle will be inserted into the lump and aspirated to remove tissue. d. You should not have anything to eat or drink for 6 hours before the procedure.

ANS: A A local anesthetic is given before stereotactic biopsy. NPO status is not needed because no sedative drugs are given. The patient is placed in the prone position. A biopsy gun is used to obtain the specimens. DIF: Cognitive Level: Apply (application) TOP: Nursing Process: Implementation MSC: NCLEX: Physiological Integrity

A female patient is receiving palliative therapy with androgen hormones as part of treatment for inoperable breast cancer. The nurse will discuss with the patient which potential body image changes that may occur as adverse effects? a. Hirsutism and acne b. Weight gain c. Flushing and hot flashes d. Alopecia and body odor A female patient is receiving palliative therapy with androgen hormones as part of treatment for inoperable breast cancer. The nurse will discuss with the patient which potential body image changes that may occur as adverse effects? a. Hirsutism and acne b. Weight gain c. Flushing and hot flashes d. Alopecia and body odor

ANS: A Androgens used for cancer treatment, such as fluoxymesterone and testolactone, can cause menstrual irregularities, virilization of female, gynecomastia, hirsutism, acne, anxiety, headache, and nausea. The patient needs to be told of these effects before therapy begins. The other options are incorrect. DIF: COGNITIVE LEVEL: Applying (Application) REF: p. 738 TOP: NURSING PROCESS: Planning MSC: NCLEX: Physiological Integrity: Pharmacological and Parenteral Therapies

A patient is receiving doxorubicin (Adriamycin) as part of treatment for ovarian cancer. Which nursing diagnosis is related to this antineoplastic drug? a. Decreased cardiac output related to the adverse effect of cardiotoxicity b. Ineffective breathing pattern related to the adverse effect of pulmonary toxicity c. Risk for injury related to the effects of neurotoxicity (ataxia, numbness of hands and feet) d. Impaired urinary elimination pattern related to hyperuricemia

ANS: A Decreased cardiac output related to the adverse effect of cardiotoxicity is a nursing diagnosis related to doxorubicin because adverse effects of doxorubicin include liver and cardiovascular toxicities. The other options are incorrect. DIF: COGNITIVE LEVEL: Applying (Application) REF: p. 742 TOP: NURSING PROCESS: Nursing Diagnosis MSC: NCLEX: Physiological Integrity: Physiological Adaptation

A 30-year-old male with HIV is diagnosed with Epstein-Barr virus. After 2 months, the virus is still active. Based upon the Epstein-Barr virus, which of the following cancers is most likely to develop in this patient? a. B-cell lymphoma b. Kaposi sarcoma c. T-cell leukemia d. T-cell lipoma

ANS: A Epstein-Barr virus is associated with B-cell lymphoma. Kaposi sarcoma is associated with HIV. Retroviruses are associated with leukemia. Lipomas are not associated with HIV.

Which of the following indicates a nurse understands a proto-oncogene? A proto-oncogene is best defined as a(n) _____ gene. a. normal b. altered c. inactive d. tumor-suppressor

ANS: A In its normal, nonmutant state, an oncogene is referred to as a proto-oncogene. A proto-oncogene is not an altered gene, an inactive gene, or a tumor-suppressor gene.

A 52-year-old male with hepatitis C recently developed hepatic cancer. Which of the following markers should be increased? a. Alpha-fetoprotein (AFP) b. Catecholamines c. Prostate-specific antigen d. Homovanillic acid

ANS: A Liver and germ cell tumors secrete a protein known as AFP, not catecholamines. Prostate tumors secrete prostate-specific antigen. Homovanillic acid is a catecholamine marker.

One of the biggest challenges facing current nursing practice is a. the number of aging Americans living with chronic disease. b. the number of patients entering into hospice programs. c. the number of cancer patients receiving supportive care. d. reduced length of stay in hospice care.

ANS: A Millions of Americans are living with one or more chronic debilitating diseases, and 7 out of 10 can expect to live with their diseases several years before dying. When coupled with the advancing age of the eight million baby boomers who now qualify for Medicare, this will soon create a huge demand on health care resources and community-based services.

A patient has a tissue growth that was diagnosed as cancer. Which of the following terms best describes this growth? a. Malignant tumor b. Lipoma c. Meningioma d. Hypertrophy

ANS: A Some tumors initially described as benign can progress to cancer and then are referred to as malignant tumors. Lipomas are benign growths, while a meningioma is a benign tumor. Hypertrophy refers to tissue overgrowth, but not cancer. REF: p. 234

The most prominent goal of palliative care is to a. integrate into chronic disease management sooner rather than later. b. enroll the patient into the Medicare Hospice Benefit. c. ensure that the patient has a 6-month prognosis. d. reserve this type of care until the patient is actively dying.

ANS: A The goal of palliative care is to integrate symptom management interventions earlier into the course of chronic disease sooner rather than later. This helps to promote optimal quality of life.

A nurse is preparing to teach the most common malignancy in children. Which malignancy should the nurse discuss? a. Leukemia b. Neuroblastoma c. Wilms tumor d. Retinoblastoma

ANS: A The most common malignancy in children is leukemia. Tumors of the nervous system, Wilms tumors, and retinoblastoma occur less frequently. REF: p. 302, Table 12-1

A 45-year-old male presents with persistent, severe stomach pain. Testing reveals a peptic ulcer. Further laboratory tests reveal the presence of Helicobacter pylori. Which of the following is of concern for this patient? a. Gastric cancer b. Leukemia c. Lung cancer d. Adenocarcinoma of the colon

ANS: A The presence of Helicobacter pylori is associated with gastric cancer, not leukemia, lung cancer, or colon cancer.

What is the effect of telomere caps on cancer cells? a. Repeated divisions b. Clonal distinction c. Limited mitosis d. Mutation abilities

ANS: A The presence of telomere caps gives cancer cells the ability to divide over and over again, thus cancer cells have unlimited mitosis. Telomere caps do not give cells clonal distinction. Mutation capability is a characteristic of cancer cells, but this property is not related to telomeres.

28. When using the accompanying illustration to teach a patient about breast self-examination, the nurse will include the information that most breast cancers are located in which part of the breast? a. 1 b. 2 c. 3 d. 4 e. 5

ANS: A The upper outer quadrant is the location of most of the glandular tissue of the breast. DIF: Cognitive Level: Understand (comprehension) TOP: Nursing Process: Planning MSC: NCLEX: Physiological Integrity

26. A patient who is scheduled for a lumpectomy and axillary lymph node dissection tells the nurse, I would rather not know much about the surgery. Which response by the nurse is best? a. Tell me what you think is important to know about the surgery. b. It is essential that you know enough to provide informed consent. c. Many patients do better after surgery if they have more information. d. You can wait until after surgery for teaching about pain management.

ANS: A This response shows sensitivity to the individual patients need for information about the surgery. The other responses are also accurate, but the nurse should tailor patient teaching to individual patient preferences. DIF: Cognitive Level: Apply (application) OBJ: Special Questions: Prioritization TOP: Nursing Process: Assessment MSC: NCLEX: Psychosocial Integrity

15. A student nurse prepares a list of teaching topics for a patient with a new diagnosis of breast cancer. Which item should the charge nurse suggest that the student nurse omit from the teaching topic list about breast cancer diagnostic testing? a. CA 15-3 level testing b. HER-2 receptor testing c. Estrogen receptor testing d. Oncotype DX assay testing

ANS: A Tumor markers such as CA 15-3 are used to monitor response to treatment for breast cancer, not to detect or diagnose breast cancer. The other tests are likely to be used for additional diagnostic testing in a patient with breast cancer. DIF: Cognitive Level: Apply (application) TOP: Nursing Process: Planning MSC: NCLEX: Safe and Effective Care Environment

A 3-year-old female was diagnosed with Wilms tumor. This disease is a tumor of the: a. kidney. b. brain. c. bone marrow. d. liver.

ANS: A Wilms tumor is a tumor found in the kidney, not the brain, the bone marrow, or the liver.

A 70-yr-old patient who has had a transurethral resection of the prostate (TURP) for benign prostatic hyperplasia (BPH) is being discharged from the hospital today. Which patient statement indicates a need for the nurse to provide additional instruction? a. "I should call the doctor if I have incontinence at home." b. "I will avoid driving until I get approval from my doctor." c. "I should schedule yearly appointments for prostate examinations." d. "I will increase fiber and fluids in my diet to prevent constipation."

ANS: A Because incontinence is common for several weeks after a TURP, the patient does not need to call the health care provider if this occurs. The other patient statements indicate that the patient has a good understanding of post-TURP instructions.

The nurse will inform a patient with cancer of the prostate that side effects of leuprolide (Lupron) may include a. flushing. b. dizziness. c. infection. d. incontinence.

ANS: A Hot flashes may occur with decreased testosterone production. Dizziness may occur with the -blockers used for benign prostatic hyperplasia. Urinary incontinence may occur after prostate surgery, but it is not an expected side effect of medication. Risk for infection is increased in patients receiving chemotherapy.

Several patients call the urology clinic requesting appointments with the health care provider as soon as possible. Which patient will the nurse schedule to be seen first? a. A 22-yr-old patient who has noticed a firm, nontender lump on his scrotum b. A 35-yr-old patient who is concerned that his scrotum "feels like a bag of worms" c. A 40-yr-old patient who has pelvic pain while being treated for chronic prostatitis d. A 70-yr-old patient who is reporting frequent urinary dribbling after a prostatectomy

ANS: A The patient's age and symptoms suggest possible testicular cancer. Some forms of testicular cancer can be very aggressive, so the patient should be evaluated by the health care provider as soon as possible. Varicoceles do require treatment but not emergently. Ongoing pelvic pain is common with chronic prostatitis. Urinary dribbling is a common problem after prostatectomy.

(Multiple Response) The nurse is caring for a patient infected with human immunodeficiency virus (HIV) who has just been diagnosed with asymptomatic chronic HIV infection. Which prophylactic measures will the nurse include in the plan of care (select all that apply)? a. Hepatitis B vaccine b. Pneumococcal vaccine c. Influenza virus vaccine d. Trimethoprim-sulfamethoxazole e. Varicella zoster immune globulin

ANS: A, B, C Asymptomatic chronic HIV infection is a stage between acute HIV infection and a diagnosis of symptomatic chronic HIV infection. Although called asymptomatic, symptoms (e.g., fatigue, headache, low-grade fever, night sweats) often occur. Prevention of other infections is an important intervention in patients who are HIV positive, and these vaccines are recommended as soon as the HIV infection is diagnosed. Antibiotics and immune globulin are used to prevent and treat infections that occur later in the course of the disease when the CD4+ counts have dropped or when infection has occurred.

MULTIPLE RESPONSE 1. Brachytherapy is being used to treat cancer in a patient. What types of cancers respond well to brachytherapy? (select all that apply) a. Prostate b. Cervix c. Head d. Neck e. Lung

ANS: A, B, C, D Radiation sources can be temporarily placed into body cavities through a delivery method termed brachytherapy. Brachytherapy is useful in the treatment of cervical, prostate, and head and neck cancers. It is not used in the treatment of lung cancer. REF: p. 260

(Multiple Response) The nurse plans a presentation for community members about how to decrease the risk for antibiotic-resistant infections. Which information will the nurse include in the teaching plan (select all that apply)? a. Antibiotics may sometimes be prescribed to prevent infection. b. Continue taking antibiotics until all of the prescription is gone. c. Unused antibiotics that are more than a year old should be discarded. d. Antibiotics are effective in treating influenza associated with high fevers. e. Hand washing is effective in preventing many viral and bacterial infections.

ANS: A, B, E All prescribed doses of antibiotics should be taken. In some situations, such as before surgery, antibiotics are prescribed to prevent infection. There should not be any leftover antibiotics because all prescribed doses should be taken. However, if there are leftover antibiotics, they should be discarded immediately because the number left will not be enough to treat a future infection. Hand washing is generally considered the single most effective action in decreasing infection transmission. Antibiotics are ineffective in treating viral infections such as influenza.

Individuals of low socioeconomic status are at an increased risk for infection because of which of the following? (Select all that apply.) a. Uninsured or underinsured status b. Easy access to health screenings c. High cost of medications d. Inadequate nutrition e. mostly female gender

ANS: A, C, D Individuals of low socioeconomic status tend to be part of the underinsured or uninsured population. Lack of insurance decreases accessibility to health care in general and health screening services specifically. High costs of medication and nutritious food also make this population at higher risk for infection.

When a patient is receiving cisplatin (Platinol-AQ) chemotherapy, the nurse will monitor for which adverse effects? (Select all that apply.) a. Tinnitus b. Heart failure c. Hearing loss d. Elevated blood urea nitrogen and creatinine levels e. Numbness or tingling in the extremities f. Elevated glucose and ketone levels

ANS: A, C, D, E Cisplatin can cause nephrotoxicity, ototoxicity, and peripheral neuropathy. Nephrotoxicity is manifested by rising blood urea nitrogen and creatinine levels; ototoxicity is manifested by tinnitus, hearing loss, and dizziness; peripheral neuropathy is manifested by numbness or tingling of the extremities. DIF: COGNITIVE LEVEL: Applying (Application) REF: p. 736 TOP: NURSING PROCESS: Evaluation MSC: NCLEX: Physiological Integrity: Reduction of Risk Potential

The nurse is assessing a patient who is receiving chemotherapy with an alkylating drug. Which assessment findings would be considered indications of an oncologic emergency? (Select all that apply.) a. Dry, "scratchy," or "swollen" throat b. Loss of hair c. Decreased red blood cell count d. White patches in the mouth or throat e. Temperature of 100.7° F (38.2° C) f. Decreased urine output

ANS: A, D, E, F Indications of an oncologic emergency include fever and/or chills with a temperature higher than 100.5° F (38.1° C); new sores or white patches in the mouth or throat; changes in bladder function or patterns; dry, burning, "scratchy," or "swollen" throat; and other signs and symptoms (see Box 46-4). The prescriber must be contacted immediately if any of the listed signs or symptoms occur. Loss of hair and decreased red blood cell count (a result of bone marrow suppression) are expected effects of chemotherapy. DIF: COGNITIVE LEVEL: Analyzing (Analysis) REF: p. 744 TOP: NURSING PROCESS: Assessment MSC: NCLEX: Physiological Integrity: Pharmacological and Parenteral Therapies

Which patient requires the most rapid assessment and care by the emergency department nurse? a. The patient with hemochromatosis who reports abdominal pain b. The patient with neutropenia who has a temperature of 101.8 F c. The patient with sickle cell anemia who has had nausea and diarrhea for 24 hours d. The patient with thrombocytopenia who has oozing after having a tooth extracted

ANS: B A neutropenic patient with a fever is assumed to have an infection and is at risk for rapidly developing sepsis. Rapid assessment, cultures, and initiation of antibiotic therapy are needed. The other patients also require rapid assessment and care but not as urgently as the neutropenic patient.

Which problem reported by a patient with hemophilia is most important for the nurse to communicate to the physician? a. Leg bruises b. Tarry stools c. Skin abrasions d. Bleeding gums

ANS: B Melena is a sign of gastrointestinal bleeding and requires collaborative actions such as checking hemoglobin and hematocrit and administration of coagulation factors. The other problems indicate a need for patient teaching about how to avoid injury, but are not indicators of possible serious blood loss.

A patient with septicemia develops prolonged bleeding from venipuncture sites and blood in the stools. Which action is most important for the nurse to take? a. Avoid venipunctures. b. Notify the patients physician. c. Apply sterile dressings to the sites. d. Give prescribed proton-pump inhibitors.

ANS: B The patients new onset of bleeding and diagnosis of sepsis suggest that disseminated intravascular coagulation (DIC) may have developed, which will require collaborative actions such as diagnostic testing, blood product administration, and heparin administration. The other actions also are appropriate, but the most important action should be to notify the physician so that DIC treatment can be initiated rapidly.

A 68-year-old woman with acute myelogenous leukemia (AML) asks the nurse whether the planned chemotherapy will be worth undergoing. Which response by the nurse is appropriate? a. If you do not want to have chemotherapy, other treatment options include stem cell transplantation. b. The side effects of chemotherapy are difficult, but AML frequently goes into remission with chemotherapy. c. The decision about treatment is one that you and the doctor need to make rather than asking what I would do. d. You dont need to make a decision about treatment right now because leukemias in adults tend to progress quite slowly.

ANS: B This response uses therapeutic communication by addressing the patients question and giving accurate information. The other responses either give inaccurate information or fail to address the patients question, which will discourage the patient from asking the nurse for information.

The nurse is planning to administer a transfusion of packed red blood cells (PRBCs) to a patient with blood loss from gastrointestinal hemorrhage. Which action can the nurse delegate to unlicensed assistive personnel (UAP)? a. Verify the patient identification (ID) according to hospital policy. b. Obtain the temperature, blood pressure, and pulse before the transfusion. c. Double-check the product numbers on the PRBCs with the patient ID band. d. Monitor the patient for shortness of breath or chest pain during the transfusion.

ANS: B UAP education includes measurement of vital signs. UAP would report the vital signs to the registered nurse (RN). The other actions require more education and a larger scope of practice and should be done by licensed nursing staff members.

The health care provider prescribes finasteride (Proscar) for a patient who has benign prostatic hyperplasia (BPH). When teaching the patient about the drug, the nurse informs him that a. he should change position from lying to standing slowly to avoid dizziness. b. his interest in sexual activity may decrease while he is taking the medication. c. improvement in the obstructive symptoms should occur within about 2 weeks. d. he will need to monitor his blood pressure frequently to assess for hypertension.

ANS: B A decrease in libido is a side effect of finasteride because of the androgen suppression that occurs with the drug. Although orthostatic hypotension may occur if the patient is also taking a medication for erectile dysfunction, it should not occur with finasteride alone. Improvement in symptoms of obstruction takes about 6 months. The medication does not cause hypertension.

22. A 36-year-old who has a diagnosis of fibrocystic breast changes calls the nurse in the clinic with symptoms. Which is most important to report to the health care provider? a. There is yellow-green discharge from the patients right nipple. b. There is an area on the breast that is hot, pink, and tender to touch. c. The lumps are firm and most are in the upper outer breast quadrants. d. The lumps are larger and painful before the patients menstrual period.

ANS: B An area that is hot or pink suggests an infectious process such as mastitis, which would require further assessment and treatment. The other information also will be reported, but these findings are typical in fibrocystic breasts. DIF: Cognitive Level: Apply (application) OBJ: Special Questions: Prioritization TOP: Nursing Process: Implementation MSC: NCLEX: Physiological Integrity

The interprofessional core team includes members from a. nursing, medicine, pharmacy, and nutrition. b. medicine, nursing, social work, and clergy. c. medicine, nursing, physical therapists, and volunteers. d. nursing, home health aides, volunteers, and clergy.

ANS: B An interprofessional team approach involving health care professionals from different disciplines is central to optimal palliative care practice and quality outcomes. The interdisciplinary core team includes members from medicine, nursing, social work, and clergy. Ancillary disciplines are also included.

An oncologist is discussing when a cancer cell loses differentiation. Which of the following is the oncologist describing? a. Autonomy b. Anaplasia c. Pleomorphic d. Metastasis

ANS: B Anaplasia, not autonomy, is the loss of differentiation. The term pleomorphic refers to a marked variability of size and shape. A malignant tumor has the ability to spread far beyond the tissue of origin by the process of metastasis.

The nurse is teaching a class about the various chemotherapy drugs. Which of these statements explains why alkylating drugs are also called "cell cycle-nonspecific drugs"? a. They are cytotoxic during a specific cell cycle. b. They are cytotoxic in any phase of the cell cycle. c. They are effective against several types of neoplasms. d. They are more highly differentiated than cell cycle-specific drugs.

ANS: B Cell cycle-nonspecific drugs kill cancer cells during any phase of the growth cycle, whereas cell cycle-specific drugs kill cancer cells during specific phases of the cell growth cycle. The other options are incorrect. DIF: COGNITIVE LEVEL: Understanding (Comprehension) REF: p. 734 TOP: NURSING PROCESS: Evaluation MSC: NCLEX: Physiological Integrity: Reduction of Risk Potential

When giving cisplatin (Platinol-AQ), the nurse is aware that the major dose-limiting effect of this drug is which condition? a. Alopecia b. Kidney damage c. Cardiotoxicity d. Stomatitis

ANS: B Cisplatin may cause nephrotoxicity, and the patient's renal function must be monitored closely while on this drug. Ensuring hydration will help to prevent nephrotoxicity. DIF: COGNITIVE LEVEL: Understanding (Comprehension) REF: p. 735 TOP: NURSING PROCESS: Implementation MSC: NCLEX: Physiological Integrity: Pharmacological and Parenteral Therapies

A nurse is giving an example of inflammation as an etiology for cancer development. What is the best example the nurse should give? a. Pneumonia and lung cancer b. Ulcerative colitis and colon cancer c. Prostatic hypertrophy and prostate cancer d. Hypercholesterolemia and leukemia

ANS: B Individuals with a 10+ year history of ulcerative colitis have a 30-fold increase in developing colon cancer. There is no relationship between pneumonia and lung cancer; between prostatic hypertrophy and cancer of the prostate; and between hypercholesterolemia and leukemia. REF: pp. 248-249

6. A 58-year-old woman tells the nurse, I understand that I have stage II breast cancer and I need to decide on a surgery, but I feel overwhelmed. What do you think I should do? Which response by the nurse is best? a. I would have a lumpectomy, but you need to decide what is best for you. b. Tell me what you understand about the surgical options that are available. c. It would not be appropriate for me to make a decision about your health. d. There is no need to make a decision rapidly; you have time to think about this.

ANS: B Inquiring about the patients understanding shows the nurses willingness to assist the patient with the decision- making process without imposing the nurses values or opinions. Treatment decisions for breast cancer do need to be made relatively quickly. Imposing the nurses opinions or showing an unwillingness to discuss the topic could cut off communication. DIF: Cognitive Level: Apply (application) TOP: Nursing Process: Implementation MSC: NCLEX: Psychosocial Integrity

4. Which assessment finding in a 36-year-old patient is most indicative of a need for further evaluation? a. Bilateral breast nodules that are tender with palpation b. A breast nodule that is 1 cm in size, nontender, and fixed c. A breast lump that increases in size before the menstrual period d. A breast lump that is small, mobile, with a rubbery consistency

ANS: B Painless and fixed lumps suggest breast cancer. The other findings are more suggestive of benign processes such as fibrocystic breasts and fibroadenoma. DIF: Cognitive Level: Apply (application) TOP: Nursing Process: Assessment MSC: NCLEX: Physiological Integrity

Palliative care used in the management of a patient with symptomatic chronic obstructive pulmonary disease (COPD) is an example of which of the following? a. Palliative care is used when the patient is beginning to die. b. Palliative care is used to help manage the symptoms that often accompany COPD. c. Hospice nurses must be involved to provide palliative care in a cancer patient. d. Patient must be enrolled into the Medicare Hospice Benefit to receive palliative care.

ANS: B Palliation is the relief or management of symptoms without providing a cure. To palliate is to reduce the severity of an actual or potential life-threatening condition or a chronic debilitating illness. Palliation is not equivalent to cure, but it is the reduction of undesirable effects resulting from the incurable disease or condition.

18. Which patient statement indicates that the nurses teaching about tamoxifen (Nolvadex) has been effective? a. I can expect to have leg cramps. b. I will call if I have any eye problems. c. I should contact you if I have hot flashes. d. I will be taking the medication for 6 to 12 months.

ANS: B Retinopathy, cataracts, and decreased visual acuity should be immediately reported because it is likely that the tamoxifen will be discontinued or decreased. Tamoxifen treatment generally lasts 5 years. Hot flashes are an expected side effect of tamoxifen. Leg cramps may be a sign of deep vein thrombosis, and the patient should immediately notify the health care provider if pain occurs. DIF: Cognitive Level: Apply (application) TOP: Nursing Process: Evaluation MSC: NCLEX: Physiological Integrity

17. Which information should the nurse include in teaching a patient who is scheduled for external beam radiation to the breast? a. The radiation therapy will take a week to complete. b. Careful skin care in the radiated area will be necessary. c. Visitors are restricted until the radiation therapy is completed. d. Wigs may be used until the hair regrows after radiation therapy.

ANS: B Skin care will be needed because of the damage caused to the skin by the radiation. External beam radiation is done over a 5- to 6-week period. Scalp hair loss does not occur with breast radiation therapy. Because the patient does not have radioactive implants, no visitor restrictions are necessary. DIF: Cognitive Level: Apply (application) TOP: Nursing Process: Implementation MSC: NCLEX: Physiological Integrity

When considering the trajectory of a specific disease, what is the most important concept? a. Hospital admissions b. Physical functioning c. Quality of life d. Symptom management

ANS: B The disease trajectory occurs from the onset of a life-limited diagnosis until death. Physical functioning determines the decline in the patient's physical status. Decline in status is used to determine when to intervene with palliative and end-of-life care.

23. The nurse notes bilateral enlargement of the breasts during examination of a 62-year-old man. Which action should the nurse take first? a. Teach the patient how to palpate the breast tissue for lumps. b. Question the patient about medications being currently used. c. Refer the patient for mammography and biopsy of the breast tissue. d. Explain that this is a temporary condition due to hormonal changes.

ANS: B The first action should be further assessment. Because gynecomastia is a possible side effect of drug therapy, asking about the current drug regimen is appropriate. The other actions may be needed, depending on the data that are obtained with further assessment. DIF: Cognitive Level: Apply (application) OBJ: Special Questions: Prioritization TOP: Nursing Process: Implementation MSC: NCLEX: Physiological Integrity

10. The nurse provides discharge teaching for a 61-year-old patient who has had a left modified radical mastectomy and lymph node dissection. Which statement by the patient indicates that teaching has been successful? a. I will need to use my right arm and to rest the left one. b. I will avoid reaching over the stove with my left hand. c. I will keep my left arm in a sling until the incision is healed. d. I will stop the left arm exercises if moving the arm is painful.

ANS: B The patient should avoid any activity that might injure the left arm, such as reaching over a burner. If the left arm exercises are painful, analgesics should be used and the exercises continued in order to restore strength and range of motion. The left arm should be elevated at or above heart level and should be used to improve range of motion and function. DIF: Cognitive Level: Apply (application) TOP: Nursing Process: Evaluation MSC: NCLEX: Physiological Integrity

11. A 33-year-old patient has a saline breast implant inserted in the outpatient surgery area. Which instruction will the nurse include in the discharge teaching? a. Take aspirin every 4 hours to reduce inflammation. b. Check wound drains for excessive blood or a foul odor. c. Wear a loose-fitting bra to decrease irritation of the sutures. d. Resume normal activities 2 to 3 days after the mammoplasty.

ANS: B The patient should be taught drain care because the drains will be in place for 2 or 3 days after surgery. Normal activities can be resumed after 2 to 3 weeks. A bra that provides good support is typically ordered. Aspirin will decrease coagulation and is typically not given after surgery. DIF: Cognitive Level: Apply (application) TOP: Nursing Process: Implementation MSC: NCLEX: Physiological Integrity

A 40-year-old female developed adenocarcinoma of the vagina. Which prenatal event is the most likely cause of her cancer? a. Rb gene mutation b. Exposure to diethylstilbestrol (DES) c. Exposure to solvents d. Exposure to radiation

ANS: B The patient with adenocarcinoma of the vagina most likely experienced prenatal exposure to DES, not an Rb gene mutation or exposure to solvents or radiation. REF: pp. 303-304

A 25-year-old male develops a tumor of the breast glandular tissue. What type of tumor will be documented on the chart? a. Carcinoma b. Adenocarcinoma c. Sarcoma d. Lymphoma

ANS: B Tumors that arise from or form ductal or glandular structures are named adenocarcinomas. Cancers arising in epithelial tissue are called carcinomas; mesenchymal tissue (including connective tissue, muscle, and bone) usually have the suffix sarcoma; lymphatic tissue are called lymphomas.

The nurse will plan to teach the patient who is incontinent of urine following a radical retropubic prostatectomy to a. restrict oral fluid intake. b. do pelvic muscle exercises. c. perform intermittent self-catheterization. d. use belladonna and opium suppositories.

ANS: B Pelvic floor muscle training (Kegel) exercises are recommended to strengthen the pelvic floor muscles and improve urinary control. Belladonna and opium suppositories are used to reduce bladder spasms after surgery. Intermittent self-catheterization may be taught before surgery if the patient has urinary retention, but it will not be useful in reducing incontinence after surgery. The patient should have a daily oral intake of 2 to 3 L.

The nurse in the clinic notes elevated prostate specific antigen (PSA) levels in the laboratory results of these patients. Which patient's PSA result is not expected to be elevated? a. A 38-yr-old patient who is being treated for acute prostatitis b. A 48-yr-old patient whose father died of metastatic prostate cancer c. A 52-yr-old patient who goes on long bicycle rides every weekend d. A 75-yr-old patient who uses saw palmetto to treat benign prostatic hyperplasia (BPH)

ANS: B The family history of prostate cancer and elevation of PSA indicate that further evaluation of the patient for prostate cancer is needed. Elevations in PSA for the other patients are not unusual.

(Multiple Response) According to the Center for Disease Control and Prevention (CDC) guidelines, which personal protective equipment will the nurse put on before assessing a patient who is on contact precautions for Clostridium difficile diarrhea (select all that apply)? a. Mask b. Gown c. Gloves d. Shoe covers e. Eye protection

ANS: B, C Because the nurse will have substantial contact with the patient and bedding when doing an assessment, gloves and gowns are needed. Eye protection and masks are needed for patients in contact precautions only when spraying or splashing is anticipated. Shoe covers are not recommended in the CDC guidelines.

When an aide asks why carcinomas rarely occur in childhood, how should the nurse respond? (select all that apply) a. Carcinomas cannot develop in an immunosuppressed environment. b. Carcinomas need a long time from exposure to occurrence. c. Carcinomas are due to environmental exposures. d. Carcinomas occur primarily in secondary sexual organs. e. Carcinomas are mesodermal.

ANS: B, C Carcinomas rarely occur in children because these cancers most commonly result from environmental carcinogens and require a long period from exposure to the appearance of the carcinoma. REF: p. 301

The nurse is caring for a patient with a diagnosed case of Clostridium difficile. The nurse expects to implement which of the following interventions? (Select all that apply.) a. Administration of protease inhibitors b. Use of personal protective equipment c. Patient teaching on methods to inhibit transmission d. Preventing visitors from entering the room e. Administration of intravenous fluids f. Strict monitoring of intake and output

ANS: B, C, E, F Protease inhibitors are used for treatment of viral infections, not bacterial infections. The nurse wants to protect visitors from exposure to the bacteria and protect the patient from secondary infection while immunocompromised, but the patient will need the support of family and close friends. Contact isolation precautions must be strictly followed along with the use of personal protective equipment and teaching on methods to inhibit transmission to help break the chain of infection. Intravenous fluids and strict intake and output monitoring will be important for the patient suffering the effects of Clostridium difficile, because it causes diarrhea with fluid loss.

Which action for a patient with neutropenia is appropriate for the registered nurse (RN) to delegate to a licensed practical/vocational nurse (LPN/LVN)? a. Assessing the patient for signs and symptoms of infection b. Teaching the patient the purpose of neutropenic precautions c. Administering subcutaneous filgrastim (Neupogen) injection d. Developing a discharge teaching plan for the patient and family

ANS: C Administration of subcutaneous medications is included in LPN/LVN education and scope of practice. Patient education, assessment, and developing the plan of care require RN level education and scope of practice.

When a patient with splenomegaly is scheduled for splenectomy, which action will the nurse include in the preoperative plan of care? a. Discourage deep breathing to reduce risk for splenic rupture. b. Teach the patient to use ibuprofen (Advil) for left upper quadrant pain. c. Schedule immunization with the pneumococcal vaccine (Pneumovax). d. Avoid the use of acetaminophen (Tylenol) for 2 weeks prior to surgery.

ANS: C Asplenic patients are at high risk for infection with Pneumococcus and immunization reduces this risk. There is no need to avoid acetaminophen use before surgery, but nonsteroidal antiinflammatory drugs (NSAIDs) may increase bleeding risk and should be avoided. The enlarged spleen may decrease respiratory depth and the patient should be encouraged to take deep breaths.

Which instruction will the nurse plan to include in discharge teaching for the patient admitted with a sickle cell crisis? a. Take a daily multivitamin with iron. b. Limit fluids to 2 to 3 quarts per day. c. Avoid exposure to crowds when possible. d. Drink only two caffeinated beverages daily.

ANS: C Exposure to crowds increases the patients risk for infection, the most common cause of sickle cell crisis. There is no restriction on caffeine use. Iron supplementation is generally not recommended. A high-fluid intake is recommended.

A 54-year-old woman with acute myelogenous leukemia (AML) is considering treatment with a hematopoietic stem cell transplant (HSCT). The best approach for the nurse to assist the patient with a treatment decision is to a. emphasize the positive outcomes of a bone marrow transplant. b. discuss the need for adequate insurance to cover post-HSCT care. c. ask the patient whether there are any questions or concerns about HSCT. d. explain that a cure is not possible with any other treatment except HSCT.

ANS: C Offering the patient an opportunity to ask questions or discuss concerns about HSCT will encourage the patient to voice concerns about this treatment and also will allow the nurse to assess whether the patient needs more information about the procedure. Treatment of AML using chemotherapy is another option for the patient. It is not appropriate for the nurse to ask the patient to consider insurance needs in making this decision.

Which finding about a patient with polycythemia vera is most important for the nurse to report to the health care provider? a. Hematocrit 55% b. Presence of plethora c. Calf swelling and pain d. Platelet count 450,000/mL

ANS: C The calf swelling and pain suggest that the patient may have developed a deep vein thrombosis, which will require diagnosis and treatment to avoid complications such as pulmonary embolus. The other findings will also be reported to the health care provider but are expected in a patient with this diagnosis.

Which action will the nurse include in the plan of care for a patient who has thalassemia major? a. Teach the patient to use iron supplements. b. Avoid the use of intramuscular injections. c. Administer iron chelation therapy as needed. d. Notify health care provider of hemoglobin 11g/dL.

ANS: C The frequent transfusions used to treat thalassemia major lead to iron toxicity in patients unless iron chelation therapy is consistently used. Iron supplementation is avoided in patients with thalassemia. There is no need to avoid intramuscular injections. The goal for patients with thalassemia major is to maintain a hemoglobin of 10 g/dL or greater.

A patient with possible disseminated intravascular coagulation arrives in the emergency department with a blood pressure of 82/40, temperature 102 F (38.9 C), and severe back pain. Which physician order will the nurse implement first? a. Administer morphine sulfate 4 mg IV. b. Give acetaminophen (Tylenol) 650 mg. c. Infuse normal saline 500 mL over 30 minutes. d. Schedule complete blood count and coagulation studies.

ANS: C The patients blood pressure indicates hypovolemia caused by blood loss and should be addressed immediately to improve perfusion to vital organs. The other actions also are appropriate and should be rapidly implemented, but improving perfusion is the priority for this patient.

A postoperative patient receiving a transfusion of packed red blood cells develops chills, fever, headache, and anxiety 35 minutes after the transfusion is started. After stopping the transfusion, what action should the nurse take? a. Draw blood for a new crossmatch. b. Send a urine specimen to the laboratory. c. Administer PRN acetaminophen (Tylenol). d. Give the PRN diphenhydramine (Benadryl).

ANS: C The patients clinical manifestations are consistent with a febrile, nonhemolytic transfusion reaction. The transfusion should be stopped and antipyretics administered for the fever as ordered. A urine specimen is needed if an acute hemolytic reaction is suspected. Diphenhydramine (Benadryl) is used for allergic reactions. This type of reaction does not indicate incorrect crossmatching.

A 16-year-old with aspirations of becoming a bodybuilder spends 3 hours a day in the gym. Five years later a hepatocellular carcinoma is discovered. Which of the following is the most likely cause of the cancer? a. Immunosuppressive agents b. Cytotoxic agents c. Anabolic steroids d. A viral infection

ANS: C A history of anabolic steroids, not immunosuppressive or cytotoxic agents, places the patient at risk for hepatocellular carcinoma. A viral infection does not place the patient at risk for hepatocellular carcinoma. REF: p. 305, Table 12-5

A patient asks when adjuvant chemotherapy is used. How should the nurse respond? Adjuvant chemotherapy treatment is used: a. as the primary treatment. b. before radiation therapy. c. after surgical removal of a tumor. d. in cancer with little risk of metastasis.

ANS: C Adjuvant chemotherapy is given after surgical excision of a cancer with the goal of eliminating micrometastases. Adjuvant chemotherapy is not given as the primary treatment or before radiation therapy. Adjuvant chemotherapy is indicated in the treatment of individuals with metastasis. REF: p. 261

5. A 53-year-old woman at menopause is discussing the use of hormone therapy (HT) with the nurse. Which information about the risk of breast cancer will the nurse provide? a. HT is a safe therapy for menopausal symptoms if there is no family history ofBRCA genes. b. HT does not appear to increase the risk for breast cancer unless there are other risk factors. c. The patient and her health care provider must weigh the benefits of HT against the risks of breast cancer. d. Natural herbs are as effective as estrogen in relieving symptoms without increasing the risk of breast cancer.

ANS: C Because HT has been linked to increased risk for breast cancer, the patient and provider must determine whether or not to use HT. Breast cancer incidence is increased in women using HT, independent of other risk factors. HT increases the risk for both nonBRCA-associated cancer and BRCA-related cancers. Alternative therapies can be used but are not consistent in relieving menopausal symptoms. DIF: Cognitive Level: Apply (application) TOP: Nursing Process: Implementation MSC: NCLEX: Physiological Integrity

The nurse is working on a plan of care with her patient which includes turning and positioning and adequate nutrition to help the patient maintain intact skin integrity. The nurse helps the patient to realize that this breaks the chain of infection by eliminating a a. host. b. mode of transmission. c. portal of entry. d. reservoir.

ANS: C Broken or impaired skin creates a portal of entry for pathogens. By maintaining intact tissue, the patient and the nurse have broken the chain of infection by eliminating a portal of entry. Host is incorrect because you are not eliminating the person or organism. Intact tissue does not eliminate the mode of transmission. Skin can still be used to transfer pathogens regardless of it being intact or broken. Intact skin does not eliminate the location for pathogens to live and grow.

A 30-year-old female is diagnosed with cancer. Testing reveals that the cancer cells have spread to local lymph nodes. A nurse realizes this cancer would be documented as stage: a. 1. b. 2. c. 3. d. 4.

ANS: C Cancer that has spread to regional structures, such as lymph nodes, is stage 3. Cancer confined to the organ of origin is stage 1. Cancer that is locally invasive is stage 2. Cancer that has spread to distant sites, such as a liver cancer spreading to lung or a prostate cancer spreading to bone, is stage 4.

A patient is receiving a third session of chemotherapy with daunorubicin (Cerubidine). The nurse will assess the patient for which signs of a potential severe toxic effect of this drug? a. Tinnitus and hearing loss b. Numbness and tingling in the fingers c. A weight gain of 2 pounds or more in 24 hours d. Decreased blood urea nitrogen and creatinine levels

ANS: C Cardiac toxicity may occur, so frequent checking of heart and breath sounds is necessary and daily weights need to be recorded (with reporting of an increase of 2 pounds or more in 24 hours or 5 pounds or more in 1 week). DIF: COGNITIVE LEVEL: Applying (Application) REF: p. 743 TOP: NURSING PROCESS: Assessment MSC: NCLEX: Physiological Integrity: Pharmacological and Parenteral Therapies

21. Which action will the nurse include in the plan of care for a patient with right arm lymphedema? a. Check blood pressure (BP) on both right and left arms. b. Avoid isometric exercise on the right arm. c. Assist with application of a compression sleeve. d. Keep the right arm at or below the level of the heart.

ANS: C Compression of the arm assists in improving lymphatic flow toward the heart. Isometric exercises may be prescribed for lymphedema. BPs should only be done on the patients right arm. The arm should not be placed in a dependent position. DIF: Cognitive Level: Apply (application) TOP: Nursing Process: Planning MSC: NCLEX: Physiological Integrity

25. The nurse is caring for a 52-year-old patient with breast cancer who is receiving chemotherapy with doxorubicin (Adriamycin) and cyclophosphamide (Cytoxan). Which assessment finding is most important to communicate to the health care provider? a. The patient complains of fatigue. b. The patient eats only 25% of meals. c. The patients apical pulse is irregular. d. The patients white blood cell (WBC) count is 5000/L.

ANS: C Doxorubicin can cause cardiac toxicity. The dysrhythmia should be reported because it may indicate a need for a change in therapy. Anorexia, fatigue, and a low-normal WBC count are expected effects of chemotherapy. DIF: Cognitive Level: Apply (application) OBJ: Special Questions: Prioritization TOP: Nursing Process: Assessment MSC: NCLEX: Physiological Integrity

2. During a well woman physical exam, a 43-year-old patient asks about her risk for breast cancer. Which question is most pertinent for the nurse to ask? a. Do you currently smoke tobacco? b. Have you ever had a breast injury? c. At what age did you start having menstrual periods? d. Is there a family history of fibrocystic breast changes?

ANS: C Early menarche and late menopause are risk factors for breast cancer because of the prolonged exposure to estrogen that occurs. Cigarette smoking, breast trauma, and fibrocystic breast changes are not associated with increased breast cancer risk. DIF: Cognitive Level: Apply (application) TOP: Nursing Process: Assessment MSC: NCLEX: Physiological Integrity

Which genetic change causes alterations in only one or a few nucleotide bases? a. Insertions b. Deletions c. Point mutations d. Amplification mutations

ANS: C Genetic changes may occur by both mutational and epigenetic mechanisms. Mutation generally means an alteration in the DNA sequence affecting expression or function of a gene. Mutations include small-scale changes in DNA, such as point mutations, which are the alteration of one or a few nucleotide base pairs. The process involved with insertions, deletions, or amplification mutations is different.

What is the nurse's priority action if extravasation of an antineoplastic drug occurs during intravenous (IV) administration? a. Reduce the infusion rate. b. Discontinue the IV, and apply warm compresses. c. Stop the infusion immediately, but leave the IV catheter in place. d. Change the infusion to normal saline, and inject the area with hydrocortisone.

ANS: C If extravasation is suspected, administration of the drug must be stopped immediately but the IV catheter left in place and the appropriate antidote instilled through the existing IV tube, after which the needle may be removed. The other options are incorrect. DIF: COGNITIVE LEVEL: Applying (Application) REF: p. 736 TOP: NURSING PROCESS: Implementation MSC: NCLEX: Safe and Effective Care Environment: Management of Care

Which information indicates a nurse understands characteristics of malignant tumors? a. Grows slowly b. Has a well-defined capsule c. Cells vary in size and shape d. Is well differentiated

ANS: C Malignant tumors have cells that vary in both size and shape, and they grow rapidly. They are poorly differentiated and not encapsulated.

What is the origin of most childhood cancers? a. Placenta b. Environment c. Mesodermal germ layer d. Neural tube

ANS: C Most childhood cancers originate from the mesodermal germ layer that gives rise to connective tissue, bone, cartilage, muscle, blood, blood vessels, the gonads, the kidney, and the lymphatic system. Most childhood cancers do not originate from the placenta or the environment. Neural tubes are generally not associated with cancer. REF: p. 301

During chemotherapy, a patient develops severe diarrhea caused by a vasoactive intestinal peptide-secreting tumor (VIPoma). The nurse expects to administer which drug for this problem? a. Dexrazoxane (Zinecard) b. Allopurinol (Zyloprim) c. Octreotide (Sandostatin) d. Bismuth subsalicylate (Pepto-Bismol)

ANS: C Octreotide (Sandostatin) is used for the management of a cancer-related condition called carcinoid crisis and treatment of the severe diarrhea caused by vasoactive intestinal peptide-secreting tumors (VIPomas). The other options are incorrect. DIF: COGNITIVE LEVEL: Applying (Application) REF: p. 740 TOP: NURSING PROCESS: Planning MSC: NCLEX: Physiological Integrity: Pharmacological and Parenteral Therapies

The nurse explains to a parent that young children diagnosed with Down syndrome are at higher risk for developing: a. nephroblastoma. b. rhabdomyosarcoma. c. leukemia. d. retinoblastoma.

ANS: C One of the more recognized syndromes is the association of trisomy 21 and Down syndrome, with an increased susceptibility to acute leukemia, not nephroblastoma, rhabdomyosarcoma, or retinoblastoma. REF: p. 303

While reviewing the complete blood count (CBC) of a patient on her unit, the nurse notes elevated basophil and eosinophil readings. The nurse realizes that this is most indicative of a _____ infection. a. bacterial b. fungal c. parasitic d. viral

ANS: C Parasitic infections are frequently indicated on a CBC by elevated basophil and eosinophil levels. Bacterial infections do not lead to elevated basophil and eosinophil levels but elevated B and T lymphocytes, neutrophils, and monocytes. Fungal infections do not lead to elevated basophil and eosinophil levels. Viral infections create elevations in B and T lymphocytes, neutrophils, and monocytes.

A nurse is teaching a group of businesspeople about disease transmission. He knows that he needs to reeducate when one of the participants states which of the following? a. "When traveling outside of the country, I need to be sure that I receive appropriate vaccinations." b. "Food and water supplies in foreign countries can contain microorganisms to which my body is not accustomed and has no resistance." c. "If I don't feel sick, then I don't have to worry about transmitted diseases." d. "I need to be sure to have good hygiene practices when traveling in crowded planes and trains."

ANS: C People can transmit pathogens even if they don't currently feel ill. Some carriers never experience the full symptoms of a pathogen. Travelers may need different vaccinations when traveling to countries outside their own because of variations in prevalent microorganisms. Food and water supplies in foreign countries can contain microorganisms that will affect a body unaccustomed to their presence. Adequate hygiene is essential when in crowded, public spaces like planes and other forms of public transportation.

When hanging a new infusion bag of a chemotherapy drug, the nurse accidentally spills a small amount of the solution onto the floor. Which action by the nurse is appropriate? a. Let it dry, and then mop the floor. b. Wipe the area with a disposable paper towel. c. Use a spill kit to clean the area. d. Ask the housekeeping department to clean the floor.

ANS: C Special spill kits are employed to clean up even the smallest chemotherapy spills. These precautions are necessary to protect the health care provider from the cytotoxic effects of these drugs. DIF: COGNITIVE LEVEL: Applying (Application) REF: p. 741 TOP: NURSING PROCESS: Implementation MSC: NCLEX: Safe and Effective Care Environment: Safety and Infection Control

8. After a 48-year-old patient has had a modified radical mastectomy, the pathology report identifies the tumor as an estrogen-receptor positive adenocarcinoma. The nurse will plan to teach the patient about a. estradiol (Estrace). b. raloxifene (Evista). c. tamoxifen (Nolvadex). d. trastuzumab (Herceptin).

ANS: C Tamoxifen is used for estrogen-dependent breast tumors in premenopausal women. Raloxifene is used to prevent breast cancer, but it is not used postmastectomy to treat breast cancer. Estradiol will increase the growth of estrogen-dependent tumors. Trastuzumab is used to treat tumors that have the HER-2 receptor. DIF: Cognitive Level: Apply (application) TOP: Nursing Process: Planning MSC: NCLEX: Physiological Integrity

Which statement indicates the patient has a correct understanding of metastasis? The most common route of metastasis is through the blood vessels and: a. lung tissue. b. body cavities. c. lymphatics. d. connective tissues.

ANS: C The most common route of metastasis is through the lymphatics, not lung tissue, body cavities, or connective tissues. REF: p. 253

The most common type of tissue cancer occurring between ages 15 and 19 is: a. sarcoma. b. squamous cell. c. carcinoma. d. neuroma.

ANS: C The most common type of tissue cancer after adolescence is carcinoma, not sarcoma, squamous cell, or neuroma. REF: p. 301

9. Which nursing action should be included in the plan of care for a patient returning to the surgical unit following a left modified radical mastectomy with dissection of axillary lymph nodes? a. Obtain a permanent breast prosthesis before the patient is discharged from the hospital. b. Teach the patient to use the ordered patient-controlled analgesia (PCA) every 10 minutes. c. Post a sign at the bedside warning against venipunctures or blood pressures in the left arm. d. Insist that the patient examine the surgical incision when the initial dressings are removed.

ANS: C The patient is at risk for lymphedema and infection if blood pressures or venipuncture are done on the right arm. The patient is taught to use the PCA as needed for pain control rather than at a set time. The nurse allows the patient to examine the incision and participate in care when the patient feels ready. Permanent breast prostheses are usually obtained about 6 weeks after surgery. DIF: Cognitive Level: Apply (application) TOP: Nursing Process: Planning MSC: NCLEX: Physiological Integrity

A 45-year-old female was recently diagnosed with cervical cancer. Which of the following is the most likely cause of her cancer? a. Herpes virus b. Rubella virus c. Human papillomavirus (HPV) d. Hepatitis B virus

ANS: C The presence of HPV is a factor in cervical cancer. The presence of herpes virus, rubella virus, or hepatitis B virus is not a factor in cervical cance

13. A patient newly diagnosed with stage I breast cancer is discussing treatment options with the nurse. Which statement by the patient indicates that additional teaching may be needed? a. There are several options that I can consider for treating the cancer. b. I will probably need radiation to the breast after having the surgery. c. Mastectomy is the best choice to decrease the chance of cancer recurrence. d. I can probably have reconstructive surgery at the same time as a mastectomy.

ANS: C The survival rates with lumpectomy and radiation or modified radical mastectomy are comparable. The other patient statements indicate a good understanding of stage I breast cancer treatment. DIF: Cognitive Level: Apply (application) TOP: Nursing Process: Assessment MSC: NCLEX: Physiological Integrity

A primary care provider is attempting to diagnose cancer and is looking for a tumor marker. Which of the following could be a possible marker? a. Red blood cells b. Apoptotic cells c. Enzymes d. Neurotransmitters

ANS: C Tumor markers include hormones, enzymes, genes, antigens, and antibodies, but not red blood cells, apoptotic cells, or neurotransmitters.

A 53-yr-old patient is scheduled for an annual physical examination. The nurse will plan to teach the patient about the purpose of a. urinalysis collection. b. uroflowmetry studies. c. prostate specific antigen (PSA) testing. d. transrectal ultrasound scanning (TRUS).

ANS: C An annual digital rectal exam (DRE) and PSA are usually recommended starting at age 50 years for men who have an average risk for prostate cancer. Urinalysis and uroflowmetry studies are done if patients have symptoms of urinary tract infection or changes in the urinary stream. TRUS may be ordered if the DRE or PSA results are abnormal.

The nurse taking a focused health history for a patient with possible testicular cancer will ask the patient about a history of a. testicular torsion. b. testicular trauma. c. undescended testicles. d. sexually transmitted infection (STI).

ANS: C Cryptorchidism is a risk factor for testicular cancer if it is not corrected before puberty. STI, testicular torsion, and testicular trauma are risk factors for other testicular conditions but not for testicular cancer.

A patient who has been recently diagnosed with benign prostatic hyperplasia (BPH) tells the nurse that he does not want to have a transurethral resection of the prostate (TURP) because it might affect his ability to have sexual intercourse. Which action should the nurse take? a. Discuss alternative methods of sexual expression. b. Teach about medication for erectile dysfunction (ED). c. Clarify that TURP does not commonly affect erection. d. Offer reassurance that fertility is not affected by TURP.

ANS: C ED is not a concern with TURP, although retrograde ejaculation is likely, and the nurse should discuss this with the patient. Erectile function is not usually affected by a TURP, so the patient will not need information about penile implants or reassurance that other forms of sexual expression may be used. Because the patient has not asked about fertility, reassurance about sperm production does not address his concerns.

To determine the severity of the symptoms for a patient with benign prostatic hyperplasia (BPH), the nurse will ask the patient about a. blood in the urine. b. lower back or hip pain. c. force of urinary stream. d. erectile dysfunction (ED).

ANS: C The American Urological Association Symptom Index for a patient with BPH asks questions about the force and frequency of urination, nocturia, and so on. Blood in the urine, ED, and back or hip pain are not typical symptoms of BPH.

A patient who has been receiving a heparin infusion and warfarin (Coumadin) for a deep vein thrombosis (DVT) is diagnosed with heparin-induced thrombocytopenia (HIT) when her platelet level drops to 110,000/L. Which action will the nurse include in the plan of care? a. Use low-molecular-weight heparin (LMWH) only. b. Administer the warfarin (Coumadin) at the scheduled time. c. Teach the patient about the purpose of platelet transfusions. d. Discontinue heparin and flush intermittent IV lines using normal saline.

ANS: D All heparin is discontinued when the HIT is diagnosed. The patient should be instructed to never receive heparin or LMWH. Warfarin is usually not given until the platelet count has returned to 150,000/L. The platelet count does not drop low enough in HIT for a platelet transfusion, and platelet transfusions increase the risk for thrombosis.

Which statement by a patient indicates good understanding of the nurses teaching about prevention of sickle cell crisis? a. Home oxygen therapy is frequently used to decrease sickling. b. There are no effective medications that can help prevent sickling. c. Routine continuous dosage narcotics are prescribed to prevent a crisis. d. Risk for a crisis is decreased by having an annual influenza vaccination.

ANS: D Because infection is the most common cause of a sickle cell crisis, influenza, Haemophilus influenzae, pneumococcal pneumonia, and hepatitis immunizations should be administered. Although continuous dose opioids and oxygen may be administered during a crisis, patients do not receive these therapies to prevent crisis. Hydroxyurea (Hydrea) is a medication used to decrease the number of sickle cell crises.

Which patient information is most important for the nurse to monitor when evaluating the effectiveness of deferoxamine (Desferal) for a patient with hemochromatosis? a. Skin color b. Hematocrit c. Liver function d. Serum iron level

ANS: D Because iron chelating agents are used to lower serum iron levels, the most useful information will be the patients iron level. The other parameters will also be monitored, but are not the most important to monitor when determining the effectiveness of deferoxamine.

Which laboratory test will the nurse use to determine whether filgrastim (Neupogen) is effective for a patient with acute lymphocytic leukemia who is receiving chemotherapy? a. Platelet count b. Reticulocyte count c. Total lymphocyte count d. Absolute neutrophil count

ANS: D Filgrastim increases the neutrophil count and function in neutropenic patients. Although total lymphocyte, platelet, and reticulocyte counts also are important to monitor in this patient, the absolute neutrophil count is used to evaluate the effects of filgrastim.

A 62-year old man with chronic anemia is experiencing increased fatigue and occasional palpitations at rest. The nurse would expect the patients laboratory findings to include a. a hematocrit (Hct) of 38%. b. an RBC count of 4,500,000/mL. c. normal red blood cell (RBC) indices. d. a hemoglobin (Hgb) of 8.6 g/dL (86 g/L).

ANS: D The patients clinical manifestations indicate moderate anemia, which is consistent with a Hgb of 6 to 10 g/dL. The other values are all within the range of normal.

A patient in the emergency department complains of back pain and difficulty breathing 15 minutes after a transfusion of packed red blood cells is started. The nurses first action should be to a. administer oxygen therapy at a high flow rate. b. obtain a urine specimen to send to the laboratory. c. notify the health care provider about the symptoms. d. disconnect the transfusion and infuse normal saline.

ANS: D The patients symptoms indicate a possible acute hemolytic reaction caused by the transfusion. The first action should be to disconnect the transfusion and infuse normal saline. The other actions also are needed but are not the highest priority.

27. The outpatient clinic receives telephone calls from four patients. Which patient should the nurse call back first? a. 57-year-old with ductal ectasia who has sticky multicolored nipple discharge and severe nipple itching b. 21-year-old with a family history of breast cancer who wants to discuss genetic testing for the BRCA gene c. 40-year-old who still has left side chest and arm pain 2 months after a left modified radical mastectomy d. 50-year-old with stage 2 breast cancer who is receiving doxorubicin (Adriamycin) and has ankle swelling and fatigue

ANS: D Although all the patients have needs that the nurse should address, the patient who is receiving a cardiotoxic medication and has symptoms of heart failure should be assessed by the nurse first. BRCA testing may be appropriate for the 21-year-old, but it does not need to be done immediately. Chest and arm pain are normal up to 3 months after mastectomy. Nipple discharge and itching is a common finding with ductal ectasia. DIF: Cognitive Level: Analyze (analysis) OBJ: Special Questions: Prioritization; Multiple Patients TOP: Nursing Process: Planning MSC: NCLEX: Safe and Effective Care Environment

19. The nurse is admitting a patient scheduled this morning for lumpectomy and axillary lymph node dissection. Which action should the nurse take first? a. Teach the patient how to deep breathe and cough. b. Discuss options for postoperative pain management. c. Explain the postdischarge care of the axillary drains. d. Ask the patient to describe what she knows about the surgery.

ANS: D Before teaching, the nurse should assess the patients current knowledge level. The other teaching also may be appropriate, depending on the assessment findings. DIF: Cognitive Level: Apply (application) OBJ: Special Questions: Prioritization TOP: Nursing Process: Implementation MSC: NCLEX: Physiological Integrity

Which information should the nurse include when teaching about angiogenic factors? In cancer, angiogenic factors stimulate: a. release of growth factors. b. tumor regression. c. apoptosis. d. new blood vessel growth.

ANS: D Cancers can secrete multiple factors that stimulate new blood vessel growth called angiogenesis, not release of growth factors or tumor regression. Apoptosis is cell death. REF: pp. 245-246

20. When the nurse is working in the womens health care clinic, which action is appropriate to take? a. Teach a healthy 30-year-old about the need for an annual mammogram. b. Discuss scheduling an annual clinical breast examination with a 22-year-old. c. Explain to a 60-year-old that mammography frequency can be reduced to every 3 years. d. Teach a 28-year-old with a BRCA-1 mutation about magnetic resonance imaging (MRI).

ANS: D MRI (in addition to mammography) is recommended for women who are at high risk for breast cancer. A young woman should have a clinical breast exam every 3 years. Annual mammograms are recommended for women over 50. DIF: Cognitive Level: Apply (application) TOP: Nursing Process: Planning MSC: NCLEX: Health Promotion and Maintenance

Today most patients are living for several years before dying with multiple chronic conditions, such as COPD, congestive heart failure, diabetes, and obesity. These concomitant diseases contribute to multiple symptoms that interfere with the patient's quality of life. What type of care would you consider for this patient? a. End-of-life care b. Supportive care c. Comfort care d. Palliative care

ANS: D Palliative care provides optimal symptom management in the setting of multiple chronic conditions. The relief and management of these symptoms help to promote improved quality of life for the patient and help to maintain physical functioning.

In order to provide the best intervention for a patient, the nurse is often responsible for obtaining a sample of exudate for culture. This test will identify a. whether a patient has an infection. b. where an infection is located. c. what cells are being utilized by the body to attack an infection. d. what specific type of pathogen is causing an infection.

ANS: D People can transmit pathogens even if they don't currently feel ill. Some carriers never experience the full symptoms of a pathogen. A CBC will identify that the patient has an infection. Inspection and radiography will help identify where an infection is located. The CBC with differential will identify the white blood cells being used by the body to fight an infection. The culture will grow the microorganisms in the sample for identification of the specific type of pathogen.

1. The nurse teaching a young womens community service group about breast self-examination (BSE) will include that a. BSE will reduce the risk of dying from breast cancer. b. BSE should be done daily while taking a bath or shower. c. annual mammograms should be scheduled in addition to BSE. d. performing BSE after the menstrual period is more comfortable.

ANS: D Performing BSE at the end of the menstrual period will reduce the breast tenderness associated with the procedure. The evidence is not clear that BSE reduces mortality from breast cancer. BSE should be done monthly. Annual mammograms are not routinely scheduled for women under age 40, and newer guidelines suggest delaying them until age 50. DIF: Cognitive Level: Apply (application) TOP: Nursing Process: Implementation MSC: NCLEX: Health Promotion and Maintenance

16. The nurse will anticipate teaching a 56-year-old patient who is diagnosed with lobular carcinoma in situ (LCIS) about a. lumpectomy. b. lymphatic mapping. c. MammaPrint testing. d. tamoxifen (Nolvadex).

ANS: D Tamoxifen is used as a chemopreventive therapy in some patients with LCIS. The other diagnostic tests and therapies are not needed because LCIS does not usually require treatment. DIF: Cognitive Level: Apply (application) TOP: Nursing Process: Planning MSC: NCLEX: Physiological Integrity

Just before the second course of chemotherapy, the laboratory calls to report that the patient's neutrophil count is 450 cells/mm3. The nurse expects that the oncologist will follow which course of treatment? a. Chemotherapy will continue as scheduled. b. Chemotherapy will resume with a lowered dosage. c. Chemotherapy will resume after a transfusion of neutrophils. d. Chemotherapy will be withheld until the neutrophil count returns toward normal levels.

ANS: D The normal range for neutrophils is above 1500 cells/mm3. If neutrophils are decreased to levels of less than 500 cells/mm3 (neutropenia), there is risk for severe infection. Chemotherapy will be held until the count returns toward normal levels. DIF: COGNITIVE LEVEL: Analyzing (Analysis) REF: p. 744 TOP: NURSING PROCESS: Evaluation MSC: NCLEX: Physiological Integrity: Pharmacological and Parenteral Therapies

Which characterizes an embryonic cancer tumor? a. Commonly occurring b. Often seen in adults c. Composed of mature, differentiated cells d. Usually manifested around age 5

ANS: D These types of cancers usually manifest around age 5. These types of cancers are not commonly occurring, seen in adults, or composed of undifferentiated cells. REF: p. 301

7. The nurse will teach a patient with metastatic breast cancer who has a new prescription for trastuzumab (Herceptin) that a. hot flashes may occur with the medication. b. serum electrolyte levels will be drawn monthly. c. the patient will need frequent eye examinations. d. the patient should call if she notices ankle swelling.

ANS: D Trastuzumab can lead to ventricular dysfunction, so the patient is taught to self-monitor for symptoms of heart failure. There is no need to monitor serum electrolyte levels. Hot flashes or changes in visual acuity may occur with tamoxifen, but not with trastuzumab. DIF: Cognitive Level: Apply (application) TOP: Nursing Process: Planning MSC: NCLEX: Physiological Integrity

The nurse will anticipate that a 61-yr-old patient who has an enlarged prostate detected by digital rectal examination (DRE) and an elevated prostate specific antigen (PSA) level will need teaching about a. cystourethroscopy. b. uroflowmetry studies. c. magnetic resonance imaging (MRI). d. transrectal ultrasonography (TRUS).

ANS: D In a patient with an abnormal DRE and elevated PSA, transrectal ultrasound is used to visualize the prostate for biopsy. Uroflowmetry studies help determine the extent of urine blockage and treatment, but there is no indication that this is a problem for this patient. Cystoscopy may be used before prostatectomy but will not be done until after the TRUS and biopsy. MRI is used to determine whether prostatic cancer has metastasized but would not be ordered at this stage of the diagnostic process.

The student demonstrates a lack of understanding of palliative care when making which statement? a. "Palliative care is designed to promote comfort." b. "Palliative care is designed to reduce disease exacerbations." c. "Palliative care is designed to decrease acute care hospital admissions." d. "Palliative care is designed to promote a cure for chronic disease."

ANS: D Palliation is the reduction of symptoms without elimination of the cause. Palliative care refers to the provision of care for patients who are diagnosed with a disease or condition without a cure.

A patient with chronic pain who has been receiving morphine sulfate 20 mg IV over 24 hours is to be discharged home on oral sustained-release morphine (MS Contin) administered twice a day. What dosage of MS Contin will be needed for each dose to obtain an equinalgesic dose for the patient? (Morphine sulfate 10 mg IV is equianalgesic to morphine sulfate 30 mg orally).

ANS: MS Contin 30 mg/dose

A patient has an order for the monoclonal antibody adalimumab (Humira). The nurse notes that the patient does not have a history of cancer. What is another possible reason for administering this drug? A. Severe anemia B. Rheumatoid arthritis C. Thrombocytopenia D. Osteoporosis

B

Abatacept (Orencia) is prescribed for a patient with severe rheumatoid arthritis. The nurse checks the patient's medical history, knowing that this medication would need to be used cautiously if which condition is present? A. Coronary artery disease B. Chronic obstructive pulmonary disease C. Diabetes mellitus D. Hypertension

B

When monitoring a patient's response to interferon therapy, the nurse notes that the major dose-limiting factor for interferon therapy is which condition? A. Diarrhea B. Fatigue C. Anxiety D. Nausea and vomiting

B

The nurse is administering a combination of three different antineoplastic drugs to a patient who has metastatic breast cancer. Which statement best describes the rationale for combination therapy? a. There will be less nausea and vomiting. b. Increased cancer-cell killing will occur. c. The drugs will prevent metastasis. d. Combination therapy reduces the need for radiation therapy.

B (Because drug-resistant cells commonly develop, exposure to multiple drugs with multiple mechanisms and sites of action will destroy more subpopulations of cells. The other options are incorrect.)

A patient, diagnosed with lymphoma, has an allergy to one of the proposed chemotherapy drugs. The tumor has not responded to other types of treatment. The nurse expects the oncologist to follow which course of treatment? a. The physician will choose another drug to use. b. The chemotherapy will be given along with supportive measures to treat a possible allergic reaction. c. The patient will receive reduced doses of chemotherapy for a longer period of time. d. The chemotherapy cannot be given because of the patient's allergy.

B (Even if a patient has a known allergic reaction to a given antineoplastic medication, the urgency of treating the patient's cancer may still necessitate administering the medication and then treating any allergic symptoms with supportive medications, such as antihistamines, corticosteroids, and acetaminophen.)

A patient is receiving high doses of methotrexate and is experiencing severe bone marrow suppression. The nurse expects which intervention to be ordered with this drug to reduce this problem? a. A transfusion of whole blood b. Leucovorin rescue c. Therapy with filgrastim (Neupogen) d. Administration of allopurinol (Zyloprim)

B (High-dose methotrexate is associated with bone marrow suppression, and it is always given in conjunction with the rescue drug leucovorin, which is an antidote for folic acid antagonists. Basically, leucovorin rescues the healthy cells from methotrexate. The other options are incorrect.)

The nurse is reviewing infection-prevention measures with a patient who is receiving antineoplastic drug therapy. Which statement by the patient indicates the need for further teaching? a. "I will avoid those who have recently had a vaccination." b. "I will eat only fresh fruits and vegetables." c. "I will report a sore throat, cough, or low-grade temperature." d. "It is important for both my family and me to practice good hand washing."

B (Patients who are neutropenic and susceptible to infections need to adhere to a low-microbe diet by washing fresh fruits and vegetables and making sure foods are well cooked. The other options are correct.)

A patient is receiving finasteride (Proscar) for treatment of benign prostatic hyperplasia. The nurse will tell him that a possible effect of this medication is: a. alopecia. b. increased hair growth. c. urinary retention. d. increased prostate size.

B Finasteride is given to reduce prostate size in men with benign prostatic hyperplasia. It has been noted that men taking this medication experience increased hair growth. The other options are incorrect.

During the administration of finasteride (Proscar), the nurse must remember which important precaution? a. It must be taken on an empty stomach. b. It must not be handled by pregnant women. c. It is given by deep intramuscular injection to avoid tissue irritation. d. The patient needs to be warned that alopecia is a common adverse effect.

B Finasteride must not be handled by pregnant women because of its teratogenic effects. It is taken orally and without regard to meals. The other options are incorrect.

A 73-year-old male patient is in the clinic for a yearly physical and is asking for a prescription for sildenafil (Viagra). He has listed on his health history that he is taking a nitrate for angina. The nurse is aware that which problem may occur if sildenafil is taken with a nitrate? a. Significant increase in pulse rate b. Significant decrease in blood pressure c. Increased risk of bleeding d. Reduced effectiveness of the sildenafil

B In patients with pre-existing cardiovascular disease, especially those on nitrates, erectile dysfunction drugs such as sildenafil lower blood pressure substantially, potentially leading to more serious adverse events. The other options are incorrect.

In addition to matching ABO antigens, a blood transfusion must also be matched for: a. HLA type. b. Rh antigen. c. immunoglobulins. d. platelet compatibility.

B Rh antigen.

During therapy with hematopoietic drugs, the nurse will monitor the patient for which adverse effects? (Select all that apply.) A. Hypotension B. Edema C. Diarrhea D. Black, tarry stools E. Nausea and vomiting F. Headache

B, C, E, F

An elderly Chinese woman is interested in biologically based therapies to relieve osteoarthritis (OA) pain. You are preparing a plan of care for her OA. Options most conducive to her expressed wishes may include which actions or activities? A. Pilates, breathing exercises, and aloe vera B. Herbs, vitamins, and tai chi C. Alternating ice and heat to relieve pain and inflammation D. Guided imagery, relaxation breathing, and meditation

B. Herbs, vitamins, and tai chi

A 19-year-old male has sustained a transection of C-7 in a motor vehicle crash rendering him a quadriplegic. He describes his pain as burning, sharp, and shooting. What type of pain is this patient describing? A. Nociceptive pain B. Neuropathic pain C. Mixed pain syndrome D. Ghost pain

B. Neuropathic pain

A patient asks about his cancer treatment with monoclonal antibodies. The nurse tells him that which is the major advantage of treating certain cancers with monoclonal antibodies? A. They will help the patient improve more quickly than will other antineoplastic drugs. B. They are more effective against metastatic tumors. C. Monoclonal antibodies target certain tumor cells and yet bypass normal cells. D. There are fewer incidences of opportunistic infections with monoclonal antibodies.

C

A patient who has received chemotherapy has a critically low platelet count. The nurse expects which drug or drug class to be used to stimulate platelet cell production? A. Filgrastim (Neupogen) B. Interferons C. Oprelvekin (Neumega) D. Epoetin alfa (Epogen)

C

A patient is to receive testosterone therapy via a transdermal patch. He asks the nurse, "Why am I getting a patch? Can't I just take a pill?" Which response by the nurse is correct? a. "The patch reduces the incidence of side effects." b. "If you don't take the patch, you will have to have injections instead." c. "The patch allows for better absorption of the medication." d. "You will only have to change the patch weekly."

C The transdermal form allows for better absorption of testosterone because of its high first-pass effect. Oral forms are poorly absorbed, and the transdermal form is preferable to an injection and is preferred for hormonal replacement therapy. The patch is changed daily.

When a nurse notices that a patient has type O blood, the nurse realizes that anti-_____ antibodies are present in the patient's body. a. A only b. B only c. A and B d. O

C A and B

When the maternal immune system becomes sensitized against antigens expressed by the fetus, what type of immune reaction occurs? a. Autoimmune b. Anaphylaxis c. Alloimmune d. Allergic

C Alloimmune

When a patient asks the nurse what hypersensitivity is, how should the nurse respond? Hypersensitivity is best defined as: a. a reduced immune response found in most pathologic states. b. a normal immune response to an infectious agent. c. an excessive or inappropriate response of the immune system to a sensitizing antigen. d. antigenic desensitization

C an excessive or inappropriate response of the immune system to a sensitizing antigen.

A 10-year-old male is stung by a bee while playing in the yard. He begins itching and develops pain, swelling, redness, and respiratory difficulties. He is suffering from: a. immunodeficiency. b. autoimmunity. c. anaphylaxis. d. tissue-specific hypersensitivity.

C anaphylaxis.

When a patient presents at the emergency department for an allergic reaction, the nurse recognizes the most severe consequence of a type I hypersensitivity reaction is: a. urticaria. b. hives. c. anaphylaxis. d. antibody-dependent cell-mediated cytotoxicity (ADCC).

C anaphylaxis.

A patient presents with poison ivy on the extremities, face, and buttocks after an initial exposure 48 hours ago. This condition is an example of: a. anaphylaxis. b. serum sickness. c. delayed hypersensitivity. d. viral disease.

C delayed hypersensitivity.

A nurse recalls that an example of an immune-complex-mediated disease is: a. bronchial asthma. b. contact dermatitis. c. serum sickness. d. rheumatoid arthritis

C serum sickness.

Controlling pain is important to promoting wellness. Unrelieved pain has been associated with which complication? A. Large tidal volumes and decreased lung capacity B. Decreased tumor growth and longevity C. Prolonged stress response and a cascade of harmful effects system-wide D. Decreased carbohydrate, protein, and fat destruction

C. Prolonged stress response and a cascade of harmful effects system-wide

Aldesleukin [IL-2] (Proleukin) is prescribed for a patient. The nurse reviews the patient's medication list and would note a potential drug interaction if which drug class is also ordered? A. Anticoagulants B. Antiepileptic drugs C. Oral hypoglycemic drugs D. Antihypertensive drugs

D

During a patient's therapy with interleukins, the nurse monitors the patient for capillary leak syndrome. Which assessment finding, if present, would indicate this problem? A. Bradycardia B. A dry cough C. Bruising on the skin D. A sudden, 15-pound weight gain

D

The nurse is developing a plan of care for a patient who is experiencing gastrointestinal adverse effects, including anorexia and nausea, after the first course of antineoplastic therapy. What is an appropriate outcome for this patient when dealing with this problem? a. The patient will eat three balanced meals a day within 2 days. b. The patient will return to normal eating pattern within 4 weeks. c. The patient will maintain normal weight by consuming healthy snacks as tolerated. d. The patient will maintain a diet of small, frequent feedings with nutrition supplements within 2 weeks.

D (Consuming small, frequent meals with nutritional supplements, and maintaining a bland diet help to improve nutrition during antineoplastic therapy.)

A patient is receiving her third course of 5-fluorouracil therapy and knows that stomatitis is a potential adverse effect of antineoplastic therapy. What will the nurse teach her about managing this problem? a. "You can take aspirin to prevent stomatitis." b. "Be sure to watch for and report black, tarry stools immediately." c. "You need to increase your intake of foods containing fiber and citric acid." d. "Be sure to examine your mouth daily for bleeding, painful areas, and ulcerations."

D (The symptoms of stomatitis consist of pain or burning in the mouth, difficulty swallowing, taste changes, viscous saliva, dryness, cracking, and fissures, with or without bleeding mucosa. Teach patients to avoid consuming foods containing citric acid and foods that are hot or spicy or high in fiber. Assessing stools is important but is not related to stomatitis, and aspirin must not be used during this therapy.)

While collecting a health history on a patient admitted for colon cancer, which of the following questions would be a priority to ask this patient? a. "Have you noticed any blood in your stool?" b. "Have you been experiencing nausea?" c. "Do you have back pain?" d. "Have you noticed any swelling in your abdomen?"

a. "Have you noticed any blood in your stool?"

A patient with chronic back pain has learned to control the pain with the use of imagery and hypnosis. The patient's spouse asks the nurse how these techniques work. Which response by the nurse is accurate? a. "The strategies work by affecting the perception of pain." b. "These techniques block the pain pathways of the nerves." c. "These strategies prevent transmission of stimuli from the back to the brain." d. "The therapies slow the release of chemicals in the spinal cord that cause pain."

a. "The strategies work by affecting the perception of pain." Cognitive therapies affect the perception of pain by the brain rather than affecting efferent or afferent pathways or influencing the release of chemical transmitters in the dorsal horn. p. 121

1. The nurse is advising a clinic patient who was exposed a week ago to human immunodeficiency virus (HIV) through unprotected sexual intercourse. The patient's antigen and antibody test has just been reported as negative for HIV. What instructions should the nurse give to this patient? a. "You will need to be retested in 2 weeks." b. "You do not need to fear infecting others." c. "Since you don't have symptoms and you have had a negative test, you do not have HIV)." d. "We won't know for years if you will develop acquired immunodeficiency syndrome (AIDS)."

a. "You will need to be retested in 2 weeks." HIV screening tests detect HIV-specific antibodies or antigens. However, there may be a delay between infection and the time a screening test is able to detect HIV. The typical "window period" for antigen-antibody combination assays is approximately 2 weeks. It is not known based on this information whether the patient is infected with HIV or can infect others. It would be best practice to have him return for repeat testing in approximately 2 weeks.

A 22-year-old was recently diagnosed with acquired immunodeficiency syndrome (AIDS). Which decreased lab finding would be expected to accompany this virus? a. CD4+ T-helper b. CD8 T-helper c. CDC cells d. CDC10 cells

a. CD4+ T-helper

16. To evaluate the effectiveness of antiretroviral therapy (ART), which laboratory test result will the nurse review? a. Viral load testing b. Enzyme immunoassay c. Rapid HIV antibody testing d. Immunofluorescence assay

a. Viral load testing The effectiveness of ART is measured by the decrease in the amount of virus detectable in the blood. The other tests are used to detect HIV antibodies, which remain positive even with effective ART.

A 76-year-old with benign prostatic hyperplasia (BPH) is agitated and confused, with a markedly distended bladder. Which intervention prescribed by the health care provider should the nurse implement first? a. insert the indwelling urinary catheter b. draw blood for a serum creatinine level c. schedule an intravenous pyelogram d. administer lorazepam 0.5 mg PO

a. insert the indwelling urinary catheter

A 72-year-old patient with a history of benign prostatic hyperplasia (BPH) is admitted with acute urinary retention and elevated blood urea nitrogen (BUN) and creatinine levels. Which prescribed therapy should the nurse implement first. a. insert urethral catheter b. obtain renal ultrasound c. draw a CBC d. infuse normal saline at 50mL/hr

a. insert urethral catheter

The nurse is caring for an adolescent patient who is dying. The patient's parents are interested in organ donation and ask the nurse how the health care providers determine brain death. Which response by the nurse accurately describes brain death determination? a. "If CPR does not restore a heartbeat, the brain cannot function." b. "Brain death has occurred if there is not any breathing or brainstem reflexes." c. "Brain death has occurred if a person has flaccid muscles and does not awaken." d. "If respiratory efforts cease and no apical pulse is audible, brain death is present."

b. "Brain death has occurred if there is not any breathing or brainstem reflexes." The diagnosis of brain death is based on irreversible loss of all brain functions, including brainstem functions that control respirations and brainstem reflexes. The other descriptions describe other clinical manifestations associated with death but are insufficient to declare a patient brain dead.

Which question asked by the nurse will give the most information about the patient's metastatic bone cancer pain? a. "How long have you had this pain?" b. "How would you describe your pain?" c. "How often do you take pain medication?" d. "How much medication do you take for the pain?"

b. "How would you describe your pain?" Because pain is a multidimensional experience, asking a question that addresses the patient's experience with the pain will elicit more information than the more specific information asked in the other three responses. All of these questions are appropriate, but the response beginning "How would you describe your pain?" is the best initial question. p. 102

The nurse is caring for a patient who has diabetes and complains of chronic, burning leg pain even when taking oxycodone (OxyContin) twice daily. Which prescribed medication is the best choice for the nurse to administer as an adjuvant to decrease the patient's pain? a. Aspirin b. Amitriptyline c. Celecoxib (Celebrex) d. Acetaminophen (Tylenol)

b. Amitriptyline The patient's pain symptoms are consistent with neuropathic pain and the tricyclic antidepressants are effective for treating this type of pain. The other medications are more effective for nociceptive pain. p. 106

A 5-year-old male becomes ill with a severe cough. Histologic examination reveals a bacterial infection, and further laboratory testing reveals cell membrane damage and decreased protein synthesis. Which of the following is the most likely cause of this illness? a. Endotoxin b. Exotoxin c. Hemolysis d. Septicemia

b. Exotoxin

What is the priority nursing diagnosis for a patient experiencing chemotherapy-induced anemia? a. Risk for injury related to poor blood clotting b. Fatigue related to decreased cellular oxygenation c. Disturbed body image related to skin color changes d. Imbalanced nutrition, less than body requirements related to anorexia

b. Fatigue related to decreased cellular oxygenation

Which of the following is a characteristic of the human immunodeficiency virus (HIV), which causes AIDS? a. HIV only infects B cells. b. HIV is a retrovirus. c. Infection does not require a host cell receptor. d. After infection, cell death is immediate.

b. HIV is a retrovirus.

A patient with prostate cancer is taking estrogen daily to control tumor growth. He reports that his left calf is swollen and painful. Which of the following would be the nurse's best action? a. Instruct the client to keep the leg elevated. b. Measure the calf circumference and compare the measurement with the right calf circumference measurement. c. Apply ice to the calf after a 10-minute massage of the area. d. Document assessment findings as an expected response with estrogen therapy.

b. Measure the calf circumference and compare the measurement with the right calf circumference measurement.

5. A pregnant woman with asymptomatic chronic human immunodeficiency virus (HIV) infection is seen at the clinic. The patient states, "I am very nervous about making my baby sick." Which information will the nurse include when teaching the patient? a. The antiretroviral medications used to treat HIV infection are teratogenic. b. Most infants born to HIV-positive mothers are not infected with the virus. c. Because it is an early stage of HIV infection, the infant will not contract HIV. d. Her newborn will be born with HIV unless she uses antiretroviral therapy (ART).

b. Most infants born to HIV-positive mothers are not infected with the virus. Only 25% of infants born to HIV-positive mothers develop HIV infection, even when the mother does not use ART during pregnancy. The percentage drops to 2% when ART is used. Perinatal transmission can occur at any stage of HIV infection (although it is less likely to occur when the viral load is lower). ART can safely be used in pregnancy, although some ART drugs should be avoided.

What common symptom should be assessed in individuals with immunodeficiency? a. Anemia b. Recurrent infections c. Hypersensitivity d. Autoantibody production

b. Recurrent infections

A cancer patient's susceptibility to the syndrome of inappropriate antidiuretic hormone (SIADH) can be suspected with which of the following laboratory results? a. Serum potassium of 5.2 mmol/L b. Serum sodium of 120 mmol/L c. Hematocrit of 40% d. Blood urea nitrogen (BUN) of 10 mg/dL

b. Serum sodium of 120 mmol/L

A patient who has fibromyalgia reports pain at level 7 (0 to 10 scale). The patient tells the nurse, "I feel depressed because I ache too much to play golf." Which patient goal has the highest priority when the nurse is developing the treatment plan? a. The patient will report pain at a level 2 of 10. b. The patient will be able to play a round of golf. c. The patient will exhibit fewer signs of depression. d. The patient will say that the aching has decreased.

b. The patient will be able to play a round of golf. For chronic pain, patients are encouraged to set functional goals such as being able to perform daily activities and hobbies. The patient has identified playing golf as the desired activity, so a pain level of 2 of 10 or a decrease in aching would be less useful in evaluating successful treatment. The nurse should also assess for depression, but the patient has identified the depression as being due to the inability to play golf, so the goal of being able to play golf is the most appropriate. p. 107

The nurse would incorporate which of the following into the plan of care as a primary prevention strategy for reduction of the risk for cancer? a. Yearly mammography for women aged 40 years and older b. Using skin protection during sun exposure while at the beach c. Colonoscopy at age 50 and every 10 years as follow-up d. Yearly prostate specific antigen (PSA) and digital rectal exam for men aged 50 and over

b. Using skin protection during sun exposure while at the beach

Which information indicates a correct understanding of viral vaccines? Most viral vaccines contain: a. active viruses. b. attenuated viruses. c. killed viruses. d. viral toxins.

b. attenuated viruses.

4. A patient who is diagnosed with acquired immunodeficiency syndrome (AIDS) tells the nurse, "I feel obsessed with morbid thoughts about dying." Which response by the nurse is appropriate? a. "Thinking about dying will not improve the course of AIDS." b. "Do you think that taking an antidepressant might be helpful?" c. "Can you tell me more about the thoughts that you are having?" d. "It is important to focus on the good things about your life now."

c. "Can you tell me more about the thoughts that you are having?" More assessment of the patient's psychosocial status is needed before taking any other action. The statements, "Thinking about dying will not improve the course of AIDS" and "It is important to focus on the good things in life" or suggesting an antidepressant discourage the patient from sharing any further information with the nurse and decrease the nurse's ability to develop a trusting relationship with the patient.

11. A patient who uses injectable illegal drugs asks the nurse about preventing acquired immunodeficiency syndrome (AIDS). Which response by the nurse is best? a. "Clean drug injection equipment before each use." b. "Ask those who share equipment to be tested for HIV." c. "Consider participating in a needle-exchange program." d. "Avoid sexual intercourse when using injectable drugs."

c. "Consider participating in a needle-exchange program." Participation in needle-exchange programs has been shown to decrease and control the rate of HIV infection. Cleaning drug equipment before use also reduces risk, but it might not be consistently practiced. HIV antibodies do not appear for several weeks to months after exposure, so testing drug users would not be very effective in reducing risk for HIV exposure. It is difficult to make appropriate decisions about sexual activity when under the influence of drugs.

Which patient should the nurse refer for hospice care? a. A 70-yr-old patient with lymphoma whose children are unable to discuss issues related to dying b. A 60-yr-old patient with chronic severe pain as a result of spinal arthritis and vertebral collapse c. A 40-yr-old patient with AIDS-related dementia who needs palliative care and pain management d. A 50-yr-old patient with advanced liver failure whose family members can no longer provide care in the home

c. A 40-yr-old patient with AIDS-related dementia who needs palliative care and pain management Hospice is designed to provide palliative care such as symptom management and pain control for patients at the end of life. Patients who require more care than the family can provide, whose families are unable to discuss important issues related to dying, or who have severe pain are candidates for other nursing services but are not appropriate hospice patients.

A middle-aged patient tells the nurse, "My mother died 4 months ago, and I just can't get over it. I'm not sure it is normal to still think about her every day." Which nursing diagnosis is most appropriate? a. Hopelessness related to inability to resolve grief b. Complicated grieving related to unresolved issues c. Anxiety related to lack of knowledge about normal grieving d. Chronic sorrow related to ongoing distress about loss of mother

c. Anxiety related to lack of knowledge about normal grieving The patient should be reassured that grieving activities such as frequent thoughts about the deceased are considered normal for months or years after a death. The other nursing diagnoses imply that the patient's grief is unusual or pathologic, which is not the case.

9. The nurse palpates enlarged cervical lymph nodes on a patient diagnosed with acute human immunodeficiency virus (HIV) infection. Which action would be appropriate for the nurse to take? a. Instruct the patient to apply ice to the neck. b. Tell the patient a secondary infection is present. c. Explain to the patient that this is an expected finding. d. Request that an antibiotic be prescribed for the patient.

c. Explain to the patient that this is an expected finding. Persistent generalized lymphadenopathy is common in the early stages of HIV infection. No antibiotic is needed because the enlarged nodes are probably not caused by bacteria. Lymphadenopathy is common with acute HIV infection and is therefore not likely the flu. Ice will not decrease the swelling in persistent generalized lymphadenopathy

A 50-year-old female experiences decreased blood pressure, decreased oxygen delivery, cardiovascular shock, and subsequent death. A complication of endotoxic shock is suspected. Which of the following is the most likely cause? a. Gram-positive bacteria b. Fungi c. Gram-negative bacteria d. Virus

c. Gram-negative bacteria

12. Which nursing action will be most useful in assisting a young adult to adhere to a newly prescribed antiretroviral therapy (ART) regimen? a. Give the patient detailed information about possible medication side effects. b. Remind the patient of the importance of taking the medications as scheduled. c. Help the patient develop a schedule to decide when the drugs should be taken. d. Encourage the patient to join a support group for adults who are HIV positive.

c. Help the patient develop a schedule to decide when the drugs should be taken. The best approach to improve adherence is to learn about important activities in the patient's life and adjust the ART around those activities. The other actions are also useful, but they will not improve adherence as much as individualizing the ART to the patient's schedule.

Which medication should the nurse administer for a patient with cancer who describes the pain as "deep, aching and at a level 8 on a 0 to 10 scale"? a. Ketorolac tablets b. Fentanyl (Duragesic) patch c. Hydromorphone (Dilaudid) IV d. Acetaminophen (Tylenol) suppository

c. Hydromorphone (Dilaudid) IV The patient's pain level indicates that a rapidly acting medication such as an IV opioid is needed. The other medications may also be appropriate to use but will not work as rapidly or as effectively as the IV hydromorphone. p. 112

6. Which patient exposure by the nurse is most likely to require postexposure prophylaxis when the patient's human immunodeficiency virus (HIV) status is unknown? a. Needle stick injury with a suture needle during a surgery b. Splash into the eyes while emptying a bedpan containing stool c. Needle stick with a needle and syringe used for a venipuncture d. Contamination of open skin lesions with patient vaginal secretions

c. Needle stick with a needle and syringe used for a venipuncture Puncture wounds are the most common means for workplace transmission of blood-borne diseases, and a needle with a hollow bore that had been contaminated with the patient's blood would be a high-risk situation. The other situations described would be much less likely to result in transmission of the virus.

An infant is experiencing hemolytic disease of the newborn. Which of the following would be found in the history and physical? a. The mother was exposed to measles. b. The father was exposed to Agent Orange. c. The baby is Rh positive. d. The baby was born 6 weeks prematurely.

c. The baby is Rh positive.

The nurse assesses that a home hospice patient with terminal cancer who complains of severe pain has a respiratory rate of 11 breaths/min. Which action should the nurse take? a. Inform the patient that increasing the morphine will cause the respiratory drive to fail. b. Tell the patient that additional morphine can be administered when the respirations are 12. c. Titrate the prescribed morphine dose up until the patient indicates adequate pain relief. d. Administer a nonsteroidal antiinflammatory drug (NSAID) to improve patient pain control.

c. Titrate the prescribed morphine dose up until the patient indicates adequate pain relief. The goal of opioid use in terminally ill patients is effective pain relief regardless of adverse effects such as respiratory depression. A nonopioid analgesic such as ibuprofen would not provide adequate analgesia or be absorbed quickly. The rule of double effect provides ethical justification for administering an increased morphine dose to provide effective pain control even though the morphine may further decrease the patient's respiratory rate. p. 125

After the nurse has received change-of-shift report, which patient should the nurse assess first? a. a patient with pneumonia who has crackles in the right lung base b. a patient with chronic bronchitis who has a low forced vital capacity c. a patient with possible lung cancer who has just returned after bronchoscopy d. a patient with hemoptysis and a 16-mm induration after tuberculin skin testing.

c. a patient with possible lung cancer who has just returned after bronchoscopy

Which information indicates a good understanding of bacterial vaccines? Most bacterial vaccines contain: a. fully active bacteria. b. synthetic bacteria. c. dead bacteria. d. bacterial toxins.

c. dead bacteria.

When the immunologist says that pathogens possess infectivity, what is the immunologist explaining? Infectivity allows pathogens to: a. spread from one individual to others and cause disease. b. induce an immune response. c. invade and multiply in the host. d. damage tissue.

c. invade and multiply in the host.

A patient with chronic neck pain is seen in the pain clinic for follow-up. To evaluate whether the pain management is effective, which question is best for the nurse to ask? a. "Has there been a change in pain location?" b. "Can you describe the quality of your pain?" c. "How would you rate your pain on a 0 to 10 scale?" d. "Does the pain keep you from activities that you enjoy?"

d. "Does the pain keep you from activities that you enjoy?" The goal for the treatment of chronic pain usually is to enhance function and quality of life. The other questions are also appropriate to ask, but information about patient function is more useful in evaluating effectiveness. p. 107

2. A patient who has a positive test for human immunodeficiency virus (HIV) antibodies is admitted to the hospital with Pneumocystis jiroveci pneumonia (PCP) and a CD4+ T-cell count of less than 200 cells/L. Based on diagnostic criteria established by the Centers for Disease Control and Prevention (CDC), which statement by the nurse is correct? a. "The patient will develop symptomatic HIV infection within 1 year." b. "The patient meets the criteria for a diagnosis of acute HIV infection." c. "The patient will be diagnosed with asymptomatic chronic HIV infection." d. "The patient has developed acquired immunodeficiency syndrome (AIDS)."

d. "The patient has developed acquired immunodeficiency syndrome (AIDS)." Development of PCP meets the diagnostic criteria for AIDS. The other responses indicate earlier stages of HIV infection than is indicated by the PCP infection.

While the nurse is obtaining the health history of a 75-year-old female patient, which of the following has the greatest implication for the development of cancer? a. Being a woman b. Family history of hypertension c. Cigarette smoking as a teenager d. Advancing age

d. Advancing age


Related study sets

Chapter 9(Controlling Microbial Growth in the Environment)

View Set

MGMT 31000: Financial Management

View Set

Chapter 15 - Principles of Management

View Set